Anda di halaman 1dari 73

Engineering Physics for High School Students

Semester 2.3. Fields

John Eggebrecht 2014

2.3.

Coulmbs Law and Gauss Theorem (2.1) F12 is the force on 1 due to 2. r12 points from the force producing point, 2, towards the field point, 1 If the product of charges q1 and q2 is negative the force is negative and negative is attractive If the magnitude of each charge is expressed in Coulombs, C, and the distance is expressed in meters then k = 9109 Nm2/C2.
E= F12 kq1 r12 = 2 q2 r12 r12

(2.2)

The field is a vector property whose magnitude and direction can be determined by placing a charge, q2, into the space surrounding the field-generating charge and measuring the resulting force. In this case the charge q2 is referred to as the test charge. The magnitude of a field is called the field strength; the field strength of E is E. Notice that there is no third law of electric fields. Consider a pair of charges. The force on one due to the presence of the other is equal and opposite to the force on the second due to the first. However, unless the charges are equal the same cannot be said of the field. In the basic CsCl (cesium chloride) crystal structure, Cs+ ions form the corners of a cube and a Cl- ion is at the cubes center. The edge length of the cube is 0.40 nm. (a) What is the magnitude of the force exerted on the Cl- ion? (b) If one of the Cs+ ions is missing what is the magnitude of the force on the Cl-? Gauss Theorem This is the most important idea in electrostatics. Unfortunately, expressed in the usual way requires integral calculus. But we can say it in words and then develop some understanding of it by looking at the consequence. Spherical Symmetry The electric field on the spherical surface with a charge Q at its center has a constant non-zero value everywhere on that surface with a direction that is perpendicular to the surface. If the surface encloses no charge then the field on that surface is zero. Cylindrical Symmetry The electric field on the cylindrical surface with a line of charge on its symmetry axis has a constant non-zero value everywhere on the surface that has a direction perpendicular to the surface. Planar Symmetry The electric field on the planar surface of a rectangular prism surrounding a plane of charge has a constant non-zero value everywhere on the surface the surfaces that are parallel to the plane of charge.

2.3.

Work Done to Assemble a Charge Distribution The potential energy of a distribution of charge depends on the separations between each pair of charges. To measure distances a coordinate system is needed. Suppose we first place a positive charge at the origin in our coordinate system. Now bring in another positive charge. The potential energy becomes more positive. We needed to push the second charge into the system against a repulsive force Positive work had to have been done on the system by the surroundings: W>0. Suppose instead that a negative charge had been added. If a winch was attached to the charge it could be used to change the surroundings, W<0. Suppose that the final and initial velocities of the positive charge that is brought from somewhere are both zero and that there are no other interactions so that there is no heating. Then the energy conservation equation !K + !U = W + Q becomes ! ! kq q !K = 0 Q = 0 " !U = W = F12 r12 = 1 2 r12 (2.3) The potential energy of a pair of charges is inversely proportional to the distance between the charges. Bring in another charge. Work must be done against the Coulomb force and the work can be expressed as sum over pairs. ! ! ! ! kq q kq q !U = F12 r12 + F13 r13 = 1 2 + 1 3 r12 r13 Every term in the sum is a scalar. Unlike the force, the potential energy is easy to evaluate. The original charge at the origin affects the surrounding space. The effect is proportional to the size of the charge. The voltage of the surrounding space is the potential energy per unit charge. kq V=! i ri (2.4) The sum is over all of the charges in the space surrounding the charge at the origin. Voltage has units of joules per coulomb. The electron-volt (eV) is the energy of an electron in a electric 5q potential of 1 volt. 1 eV = 1.610-19 J. Example Express the potential energy of the system of charges at the right in eVs if q=7 Coulomb and d=3mm.
d x d d 5q -5q d -5q

" 5q 5q 5q 5q % 5q 5q 2 9 109 Nm 2 / C2 5 49 10!12 V = k# ! + ! & = k (U=k = = 735J d d 2d ' 2d 2d 310!3 m $d

2.3.

C, but at P it is not zero and is directed outward. P, but at C it is not zero. o at either point.

67.capacitors A square loop of wire ofis resistance Rof and side a is oriented with its plane perpendicular to a magnetic field B, as apacitance of several in parallel the sum the individual shown above. What must be the rate of change of the magnetic field in order to produce a current I in the loop? or which of the following reasons? 2 2 AP (A) E Field Potential Problems IR/a Ia /R shown (C) Ia/R (D)is Ra/I (E) 22.and In (B) the circuit above, what value ofIRa the potential difference between points X and Y if the 6-volt on each capacitor depends on its capacitance, but the potential the difference Make your selection with color.

battery has no internal resistance?

(A) 1 V

(B) 2 V

(C) 3 V

( D) 4 V

(E) 6V

velocity. Which the following is charge true between the two points? is the same on each capacitor, but the potential difference across each capac68. The diagram diagram above shows anof isolated, positive Q. Point (B) isPoint twice as away from Q as pointfrom A. The 1. The above shows an isolated, positive charge Q. B far is twice as far away Q (A) Thefield resistance is at one ohm. (B) The current is one ampere. ratio of the electric strength point A to the electric field strength at point B is

23. One joule of work is needed to move one coulomb of charge from one point to another with no change in

as point A. ratio of electric field strength at point A to the electric strength at charge point (C) The difference iscentimeters one volt. The electric field strength is field oneforce newton per on coulomb. 8 isolated to 1The (B) 4 potential to 1the (C) 2 to 1 (D) 1 to (D) 1apart. (E) 1 2 54. (A) Two charges, + q and - 2q, are 2 If F to is the magnitude of the acting n its capacitance. B is-2Q, what are (E)the The electric field strength is one joule per electron. magnitude and direction of the force acting on charge + q ? capacitance is always greater than the capacitance. (A) 8 distance to 1 largest (B) 4 to 1 thin (C) 2is to 1 than(D) 1 the to 1 (E) of 1 to 69. IfMagnitude the object for a converging lens more twice focal length the2 lens, the image is Direction in a circuit always combine like resistors in series. Questions 24-25 (A) virtual and erect (B) larger than the object (C) located inside the focal point (A) (1/2) F Toward charge - 2q combination increases(D) the effective separation of the plates. located at a distance between f charge and 2f from (E) located at a distance more than 2f from the lens (B) 2 F Away from -2q the lens
a second wire

(C) F Toward charge - 2q (D) F Away charge - 2q of ength L and radius r has a resistance R. What isfrom the resistance (E) 2F Toward charge same material that has a length L/2 and a radius r/2 ? - 2q

70. Two conducting spheres of different radii, as shown above, each have charge -Q. Which of the following occurs when the two spheres are connected with a conducting wire? Two positive charges of magnitude are each each a d from the origin A of aA coordinate system as shown above. 2. Two positive charges of magnitude qq are adistance distance d from the origin of a coordinate ing efficiency of a 0.5 A, V electric (A)120 No charge flows.motor that lifts a 9 kg mass against system asmost shown above. which of the following points is the electric potential inof verage velocity of 0.5 m/s nearly (B) is Negative charge flows At from the larger sphere to the smaller sphere until the electric field atgreatest the surface 24. At which of the following points is the electric field least in magnitude? magnitude? each sphere is the same. 13% c. 25% d. 53% 75B % (C) C (A) A e. (B) (D) D (E) E (C) Negative charge flows from the larger the C smaller sphere until potential (A) A (B) B sphere to(C) (D) Dthe electric(E) E of each sphere is the same. 25. At which of following is the electric potential greatest in magnitude? 55. Light leaves a source at the X and travels points to Y along the path shown above. Which of the following statements is (D) Negative charge flows from the smaller sphere to the larger sphere until the electric field at the surface of (B) B (C)-Q C and (D) D are located (E) E on the x- and y-axes, respectively, each correct? (A) A 3. Charges +Q each sphere is the same. (A) The index of refraction is the same the two media. at afrom distance dfor from theto origin O, as shown What is the direction of (E) Negative charge flows the smaller sphere the larger sphere untilabove. the electric potential of each (B) Light travels fasterthe in medium 2 than in medium 1. O? electric field at the origin sphere is the same. (C) Snell's law breaks down at the interface. (D) Light would arrive at Y in less time by taking a straight line path from X to Y than it does taking the path shown above. (A) (B) (C) (D) (E) (E) Light leaving a source at Y and traveling to X would follow the same path shown above, but in reverse.

2R

c. R

d. R/2

e. R/4

Q are located on the56. x- A and y-axes, respectively, atis aand distance d If from the -3 coulomb. nucleus of tritium contains 2each neutrons I proton. the nucleus undergoes beta decay, emitting an electron, 4. A point P 0.50 meter from a point charge of 5.0 X 10 The n above. the nucleus is transmuted into intensity of the electric field at point P is most nearly
(A) the nucleus of an isotope of helium (B) the nucleus of an isotope of lithium (C) an alpha particle

-8 N/C 1 N/C (C) 9.0x102 N/C (D) 1.8x103 N/C (E) 7.5x108 N/C a triton a deuteron (A) (D) 2.5x10 (B) 2.5x10 e direction of the electric field at the(E) origin O?

Copyright 2004 by College Entrance Examination Board. All rights reserved. ntral.collegeboard.com (for AP professionals) and www.collegeboard.com/apstudents (for as AP shown students and parents) The 57. Charges + Q and - 4Q are situated above.

5. Charges + Q and - 4Q are situated as shown above. net electric field is which zero nearest net The electric field is zero nearest point? which (A) A (B) B (C) C (D) D (E) E point? (A) A (B) B (C) C (D) D (E) E 6. Two isolated charges, +q and -2q, are 2 centimeters apart. If F is the magnitude of the force acting on charge -2Q, what are the magnitude and direction of the force acting on charge +q?

Copyright 1993 by College Entrance Examination Board. All rights reserved. College Board, Advanced Placement Program, AP, SAT, and the acorn logo are registered trademarks of the College Entrance Examination Board.

Magnitude Direction (A) (1/2) F Toward charge - 2q (B) 2 F Away from charge -2q (C) F Toward charge - 2q (D) F Away from charge - 2q 58. 2F When an object of weight W is suspended from the center of a massless string as shown above. the tension at (E) Toward charge - 2q
any point in the string is (A) 2Wcos! (B) !Wcos! (C) Wcos! (D) W/(2cos!) (E) W/cos!
4

2.3.

The Distribution of Charge Electrons are especially mobile in metals. These materials are conductors. Electrons distribute uniformly on a conducting surface to minimize the energy of the total energy. Surface charge distribution, !. has units of C/m2. A conductor that is in contact with second conductor with larger charge capacity is grounded. An ungrounded conductor is insulated. The material surrounding an ungrounded conductor is an insulator. Excess electrons embedded in an insulator are in fixed positions. Electrons can be uniformly distributed throughout an insulator. Volume charge distribution, ". has units of C/m3 . Sometimes charge distributions are modeled as one-dimensional with a linear charge density, #. # has units of C/m. Examples 1. Three circular non-conducting rods of the same radius of curvature have uniform charges. Rod A has charge +2Q and subtends an arc of 30, rod B has charge +6Q and subtends 90, and rod C has charge +4Q and subtends 60. Rank the rods according to the linear charge density, greatest first. Lengths of the rods are LB=2LA and L=3LA. Charges are qB=3qA and qC=2qA. So #B=3qA /2LA =3/2 #A and #C=2qA /3LA =2/3 #A. Then #B > #A > #C 2. Conductors and nonconductors are very different where charge distribution is concerned. There can be no free charge within a conductor at equilibrium. Conductors behave as they do because of the mobility of charge carrying electrons. Protons are much less mobile and vibrate about fixed positions. In a volume containing a few atoms there is no net charge. This is the idea called electroneutrality: the interior of a conductor is electrically neutral. Suppose that there was an excess electron within that space. The random movement of protons and other electrons these charges produces a randomly fluctuating Coulomb force on the excess electron. This force causes it to accelerate and on average the force will cause the electron to move in the direction of minimum energy. The energy is minimized when the excess charge lies on the surface where there are fewer charges to interact with. Since there is no free charge in the interior of a conductor the average net force at all points in the interior vanishes. So, very importantly, there is no electric field within a conductor. 3. There can be no electric field in the plane of a conducting surface. This has two important consequences: 1) the electric field is always perpendicular to the surface and ii) everywhere on the conducting surface the electric potential has the same value.

2.3.

IC. AP Problems Connecting Charge Density and Field Problems 1. Identical conducting spheres 1 and 2 have equal charges and are separated by a distance that is large compared with their diameters. The electrostatic force acting on sphere 2 due to sphere 1 is F. Suppose now that a third identical sphere 3, having an insulating handle and initially neutral, is touched first to sphere 1, then to sphere 2 and finally removed. In terms of magnitude F, what is the magnitude of the electrostatic force, F that now acts on sphere 2.

wire of resistance R and side a is oriented with its plane perpendicular to a magnetic field B, as t must be the rate of change of the magnetic field in order to produce a current I in the loop? a2/R (C) Ia/R (D) Ra/I (E) IRa

2. The hollow metal sphere shown at the right is positively charged. Point C is the center of the sphere and point P is any other point within the sphere. Which of the e shows an isolated, positive charge is Q.true Point (B) is twice as far away Q points? as point A. The following of the electric field atfrom these field strength at point (A) A to It the electric field strength at point B is is zero at both points. 4 to 1 (C) 2 to 1 (D) 1 to 1 to 2 15. The hollow metal sphere shown above is positively charged (B) It is zero at(E) C,1but at P it is not zero and is directed inward. sphere and point P is any other point within the sphere. Which (C) It is zero at C, but at P it is not zero and is directed outward. e for a converging thin lens is more than twice the focal length of the lens, the image is electric field at these points? (D) It is zero P, but at C it focal is not zero. ct (B) larger than the object (C) at located inside the point It is zero Itthe is lens not zero atlocated eitherat point. tance between f and 2f(E) from (E) a distance more than 2fa. from the lens at both points. I. The rate at which each would fall freely b. It is zero at C, but at P it is not zero and is directed inward. II. The amount of mass each would balance on an equal-arm balance c. of It is zero at C, butas at shown P it is not zero and is directed outward. 3.of Two conducting spheres different radii, III. The amount momentum each would acquire when given a certain impulse at the left, each have d. It is zero at P, but at C it II, is not zero. charge Which of the (D) following occurs the two spheres are (A) I only (B) III only -Q. (C) I and II only II and III only when (E) I, and III e. It is not zero at either point. connected with a conducting wire? heres of different radii, as shown above, each have charge Which the following 57. The five resistors shown-Q. below have of the lengths and cross-sectional areas indicated and are made of material (A) No charge flows. wo spheres are connected with a conducting wire?resistivity. Which has the16. with the same greatest The resistance? total capacitance of several capacitors (B) Negative charge flows from the larger sphere to the smaller sphere until the electric fieldin atparallel is the ws. capacitances for which of the following reasons? theto surface of each sphere iselectric the same. ge flows from the larger sphere the smaller sphere until the field at the surface of a. The charge on each capacitor depends its capacitance, bu (C) Negative charge flows from the larger sphere to the smaller sphere until the electricon potential the same. ge flows from the larger sphere to the sphere smaller sphere the electric potential across of each each is of each is the until same. the same. me. (D) Negative charge flows the smaller to the larger sphere untilorbits. the electric field at laws of 58. According to the Bohrfrom model of the atom, sphere electrons orbit the nucleus in definite According to the ge flows from the smaller sphere to the larger sphere until the electric field at the surface of b. The charge is the same on each capacitor, but the potential d classical this model would be impossible because the surface ofphysics, each sphere is the same. the same. itor (A) the positively charged attracts the electrons (B) Coulomb's law applies (E) Negative charge flows from nucleus the smaller sphere to the larger sphere until the electric potential e flows from the smaller sphere to (C) the accelerating larger sphere until the radiate electricenergy potential of each electrons (D) there on is a its centripetal force on the electrons depends capacitance. of each sphere is the same. me. (E) angular momentum is conserved c. Equivalent capacitance is always greater than the largest cap d. Capacitors in a circuit always combine like resistors in serie e. The parallel combination increases the effective separation o

17. to A+2Q wireand of length L and radius r has resistance R. What is t 4. Two identical conducting spheres are charged -Q. respectively, and areaseparated 59. Two d identical spheres charged to spheres) +2Q andthe -Q. respectively, and that are separated by a distance d a radius from material has a length L/2 and by a distance (muchconducting greater than theare radii of made the assame shown above. The magnitude of (much greater than the radii of the spheres) as shown above. The magnitude of the force of attraction e. on the left a. 4R b. 2R c. R d. R/2 R/4 the force of attraction on the left sphere is F1. After the two spheres are made to touch and then sphere is F1. After the two spheres are made to touch and then are reseparated by distance d, the magnitude of are re-separated d, the magnitude of the force on the left sphere is F2. Which of the the forceby on distance the left sphere is F 2. Which of the following relationships is correct? 18. The operating efficiency following(A) relationships is correct? 2F1 = F2 (B) F1 = F2 (C) F1 = 2F2 (D) Fl=4F (E) F1 = 8 F2 of a 0.5 A, 120 V electric motor th 2 gravity at an average m/s is most nearly (A) 2F1=F2 (B) F1= F2 (C) F1=2 F2 (D) F1=4 velocity F2 (E)of F10.5 =8 F 2 a. 7% b. 13% c. 25% d. 53% e. 75 % Questions 19-20

2.3.
Copyright 1993 by College Entrance Examination Board. All rights reserved.

Charges -Q and +Q are located on the x- and y-axes, respectively, e

5. A solid conducting sphere is given a positive charge Q. How is the charge Q distributed in or on the sphere? (A) It is concentrated at the center of the sphere. (B) It is uniformly distributed throughout the sphere. (C) Its density decreases radially outward from the center. (D) Its density increases radially outward from the center. (E) It is uniformly distributed on the surface of the sphere only. 6. A positive charge of 10-6 coulomb is placed on an insulated solid conducting sphere. Which of the following is true? (A) The charge resides uniformly throughout the sphere. (B) The electric field inside the sphere is constant in magnitude, but not zero. (C) The electric field in the region surrounding the sphere increases with increasing distance from the sphere. (D) An insulated metal object acquires a net positive charge when brought near to, but not in contact with, the sphere. (E) When a second conducting sphere is connected by a conducting wire to the first sphere, charge is transferred until the electric potentials of the two spheres are equal.

2.3.

IC. AP Problems Energy, Work and Field of Charge Distributions 1. One joule of work is needed to move one coulomb of charge from one point to another with no change in velocity. Which of the following is true between the two points? (A) The resistance is one ohm. (B) The current is one ampere. (C) The potential difference is one volt. (D) The electric field strength is one newton per coulomb. (E) The electric field strength is one joule per electron. 2. The electron volt is a measure of (A) charge (B) energy (C) impulse (D) momentum (E) velocity

3. A point P is 0.50 meter from a point charge of 5.0 X 10-3 coulomb. The intensity of the electric field at point P is most nearly (A) 2.5x10-8 N/C (B) 2.5x101 N/C (C) 9.0x102 N/C (D) 1.8x103 N/C (E) 7.5x108 N/C 4. The electric potential at point P is most nearly (A) 2.5xl0-8 V (B) 2.5x101 V (C) 9.0x102 V (D) 1.8x103 V (E) 7.5x103 V

5. The figure shows two particle, each with a charge of +Q, located at the opposite corners of a square of side d. What is the direction of the net electric field at point P? (A) (B) (C) (D) (E)

6. What is the potential energy of a particle of charge +q that is held at point P? (A)
2 qQ 4!" 0 d

(B)

1 qQ 4!" 0 d

(C)

2 qQ 4!" 0 d

(D) 2 2 qQ
4!" 0 d

(E) zero

2.3.

SL Equipotentials and Field Lines A contour map of a landscape is made by determining elevations at many points and then connecting points at the same elevation with a curve. All points on the same line are at the same elevation. So all of the points on a contour have the same gravitational potential, gh. Along a contour the potential energy of interaction of an object with mass m and the Earth is mgh. The contours are called equipotentials. The collection of curves is called an equipotential surface. An equipotential surrounding a charge is a set of points on which the electric potential has the same value. The equipotentials are spherical shells centered on the charge. At each point on the equipotential surface the electric field vector is perpendicular to the surface. The field surrounding a distribution of charge is described by a set of electric field lines and a set of equipotential surfaces that are everywhere perpendicular to every field line. You can see what that means by measuring electric potential to map the field of a charge distribution. Connect two sides a battery to two rings of silver painted onto a piece of conducting paper. The potential difference between the two rings is the same as the potential difference between the battery terminals. With a 9V battery the +ve ring is at an elevation of 9V relative to the elevation of the ve ring. At every point on the paper the electric potential has some value between these. Connect all of the points with the same electric potential and you have a set of equipotentials. You can find the equipotentials by finding pairs of points between which the potential difference reading on a voltmeter is zero. Mark this by connecting them with a line on the gridded graph paper. Once your have constructed a sufficient set of equipotentials on the paper, construct the field lines that are everywhere perpendicular to every equipotential. Do this process for two symmetries. Include in your report a summary of your process, the original field maps, and the predicted field map for a charge distribution whose potential surfaces you have not measured. Support your prediction using evidence of data collected from measured field maps.

2.3.

2011 AP PHYSICS B FREE-RESPONSE QUESTIONS

2011 AP PHYSICS B FREE-RESPONSE QUESTIONS AP Field Problem 2. (15 points) An isolated, solid copper sphere of radius R1 =0.12 m has a positive charge of 6.4 10-9 C. 2. (15 points) solid copper sphere of radius R 0.12 m has a positive charge of 6.4 10 9 C . An isolated, (a) 1 9 i. Calculate the electric potential at a point 0.10 m from the center of the sphere. An (a) isolated, solid copper sphere of radius R1 0.12 m has a positive charge of 6.4 10 C . ii. Calculate the electric potential at a point 0.24 m from the center of the sphere.
(a)
i. Calculate the electric potential at a point 0.10 m from the center of the sphere. ii. i. Calculate the electric potential at a point 0.24 0.10 m from the center of the sphere. (b) On axes below, sketch a graph ofat electric V versus radiusof r from the center of the sphere. Label ii.the Calculate the electric potential a pointpotential 0.24 m from the center the sphere. (b)the Onvalue the axes sketch a graph 0 on the vertical axis. of electric potential V versus radius r from the center of at r below, (b) the axes below, potential V versus radius r from the center of the sphere. Label theOn sphere. Label the sketch value a atgraph r= 0 of onelectric the vertical axis. the value at r 0 on the vertical axis.

(c) (c)
(c)

i. Determine the magnitude of the electric field at a point 0.10 m from the center of the sphere. i. Determine the magnitude of the electric field at a point 0.10 m from the center of the sphere. ii. Determine the magnitude of the electric field at a point 0.24 m from the center of the ii. i. Determine the magnitude of the electric field at a point 0.24 0.10 m from the center of the sphere. sphere.

ii. Determine the magnitude of the electric field at a point 0.24 m from the center of the sphere. (d) A second copper sphere of radius R2 that is uncharged is placed near the first sphere, as represented in the figure below. On sphere the axes a is graph of electric potential Vthe versus along the x-axis R2 that uncharged is placed near first distance sphere, as represented in the (d) A second copper ofbelow, radius sketch (d)shown, A second copper sphere of radius R2 that isxuncharged is placed near the first sphere, as where the center of the first sphere is at = 0. figure below. On the axes below, sketch a graph of electric potential V versus distance along the x-axis represented in the figure On sphere the axes below, shown, where the centerbelow. of the first is at x = 0. sketch a graph of electric potential V versus

distance along the x-axis shown, where the center of the first sphere is at x = 0.

2.3.

2011 The College Board. Visit the College Board on the Web: www.collegeboard.org. 2011 The College Board. Visit the College Board on the Web: www.collegeboard.org. GO

10

ON TO THE NEXT PAGE.

-7-

GO ON TO THE NEXT PAGE.

AP Design Problem You are to determine the magnitude and direction of the electric field at a point between two large parallel conducting plates. The two plates have equal but opposite charges, but it is not known which is positive and which is negative. The plates are mounted vertically on insulating stands. (a) A small ball of known mass m, with a small charge +q of known magnitude, is provided. The ball is attached to an insulating string. The additional laboratory equipment available includes only those items listed below, plus stands and clamps as needed. Choose the equipment you would use to make measurements needed to determine the magnitude and direction of the electric field between the two plates. ____ Wooden meterstick ____ Protractor ____ Screen ____ Spring scale ____ Stopwatch ____ Bright light ____ Metal rod ____ Camera (still or video) ____ Binoculars

(b) Sketch a diagram of the experimental setup and label the pieces of equipment used.

(c) Outline the experimental procedure you would use, including a list of quantities you would measure. For each quantity, identify the equipment you would use to make the measurement.

(d) i. Explain how you would calculate the magnitude of the electric field. ii. Explain how you would determine the direction of the electric field. iii. Explain how you would determine which plate is positive.

2.3.

11

Field Symmetry The equipotentials of a charge distribution have the same symmetry as the distribution. For this course these symmetries are the point, the line, and the plane. Each symmetry describes how the strength of the field due to the source decays as distance to the source increases. A point charge is a source and the strength of the field due to the source decays like 1/4!r2. The equipotential surfaces are spherical shells with radius r and area 4!r2. q E= ! 4" r 2 A line charge is a source per unit length, q/L, and the strength of the field due to the source decays like 1/2!r. The equipotential surfaces are cylindrical shells with radius r and area 2!rL (2.5) A charged plate is a source per unit area, q/A, and the strength of the field due to the source is constant. The equipotential surfaces are planes with area A.
E= q !A

(2.6)

Each of these electric fields can be expressed as q/$A. The permittivity of the space in which the fields lie is a constant property of the medium within the space. The permittivity of free space is a fundamental constant, $0, where $0=1/(4!k) = 8.8510-12 C2/Nm2 = 8.85 pF/m. This result is one of the principle organizing principles of physics. It was obtained by Carl Friedrich Gauss in the mid-19th century. It is the only principle to date that connects the force laws for mass and charge. Graphs of the dependence of field strength on distance from the source, x, for these three symmetries show that the field strength decays most rapidly for the point charge and does not decay at all for the plane of charge.

2.3.

12

IC. Electric Field Lines The convention is that electric field lines originate on positive charges and terminate on negative charges. 1. Draw some electric field lines and equipotentials for these charge distributions: A. An negative point charge

B. A pair of opposite charges

C. A pair of oppositely charged planes

D. A pair of oppositely charged disk made by flattening a pair of point charges.

2.3.

13

64. A wire loop is rotated in a uniform magnetic field about an axis perpendicular to the field. as shown above. How many times is the induced current in the loop reversed if the loop makes 3 complete revolutions from the position shown? a. One b. Two c. Three d. Six e. Twelve 65. A particle of charge Q and mass m is accelerated from rest through a potential difference V, 2. A hollow metal sphere 1.0 m in diameter carries a charge of 4.0 C. The electric field at a attaining a kinetic energy K. What is the kinetic energy of a particle of charge 2Q and mass distance of 2.0 m from the center of the sphere is most nearly m/2 that potential difference? 3 is accelerated from 4 rest through the same 4 (A) 9.0x10 N/C (C) (D) 3.6x104 N/C (E) 1.4x105 N/C a. 4 b. 2(B) 1.8x10 c. K N/C d. 2K2.4x10e. N/C 4K

3. The diagram above shows electric field lines in an isolated region of space containing two 66. The diagram aboveY shows electric lines in an isolated region of space containing two small charged spheres, and Z Whichfield of the following statements is true? small charged Y andand Z Which of the following statements is true? (A) The charge on spheres, Y is negative the charge on Z is positive. a. The charge on Y is negative and the charge on Z is positive. (B) The strength of the electric field is the same everywhere. b. The strength of electricmidway field is the same Y everywhere. (C) The electric field isthe strongest between and Z. c. small The electric field is strongest midway Z. tend to move toward the right. (D) A negatively charged object placedbetween at point Y X and would d. A small negatively charged object placed at point X would (E) Both charged spheres Y and Z carry charge of the same sign. tend to move toward the right. e. Both charged spheres Y and Z carry charge of the same sign. 4. A hollow metal sphere of radius R is positively charged. Of the following distances from the 67. of Athe satellite of which mass m and speed moves a stable, circular orbit around a planet of center sphere, location willvhave thein greatest electric field strength? mass M . What is the radius of the satellite's orbit? (A) 0 (center of the sphere) (B) 3R/2 (C) 5R/4 (D) 2R GM Gv GM is of constant GmM GmM (E) None of the above because the field strength a. b. c. 2 d. e. 2
mv mM
v v v

Copyright 2004 by College Entrance Examination Board. All rights reserved. Visit apcentral.collegeboard.com (for AP professionals) and www.collegeboard.com/apstudents (for AP students and parents)

2.3.

14

2009 AP PHYSICS B FREE-RESPONSE QUESTIONS 2009 AP PHYSICS B FREE-RESPONSE QUESTIONS

2009 AP PHYSICS B FREE-RESPONSE QUESTIONS

5. points) Two small objects, labeled 1 and 2 in the diagram above, are suspended in equilibrium from 2. (10 2. (10 points) strings of length L. Each object has mass m and charge +Q. Assume that the strings have Two small objects, labeled and 2 in the diagram above, are suspended in equilibrium from strings of length L. negligible mass and are 1 insulating and electrically Express algebraic answers inof terms Two small objects, labeled 1 and 2 in the diagram above,neutral. are suspended in all equilibrium from strings length L. Each object has mass m and charge +Q. Assume that the strings have negligible mass and are insulating and m and chargeconstants. +Q. Assume that the strings have negligible mass and are insulating and Each object has of m , L, Q, q, mass and fundamental electrically neutral. Express all algebraic answers in terms of m, L, Q, ! , and fundamental constants. electrically neutral. Express all algebraic answers in terms of m, L, Q, ! , and fundamental constants. (a) following diagram, sketch lines to illustrate a 2-dimensional view of the net electric field due to the (a)On Onthe the following diagram, sketch lines to illustrate a 2-dimensional of electric the net field electric (a) On the following diagram, sketch lines to illustrate a 2-dimensional view ofview the net due to the two objects in the region enclosed by the dashed lines. field due to the objects in the region enclosed by the dashed lines. two objects in two the region enclosed by the dashed lines. 2. (10 points)
Two small objects, labeled 1 and 2 in the diagram above, are suspended in equilibrium from strings of length L. Each object has mass m and charge +Q. Assume that the strings have negligible mass and are insulating and electrically neutral. Express all algebraic answers in terms of m, L, Q, ! , and fundamental constants. (a) On the following diagram, sketch lines to illustrate a 2-dimensional view of the net electric field due to the two objects in the region enclosed by the dashed lines.

(b) Derive an expression for the electric potential at point A, shown in the diagram at the top of the page, which (b) Derive an expression for the electric potential at point A, shown in the diagram at the top of the page, which midway between the charged objects. (b)is an expression for theobjects. electric potential at point A, shown in the diagram at the top of isDerive midway between the charged the page, which is midway between the charged objects. (c) On the following diagram of object 1, draw and label vectors to represent the forces on the (c) On the following diagram of object 1, draw and label vectors to represent the forces on the object. object.

(b) Derive an expression for the electric potential at point A, shown in the diagram at the top of the page, which is midway between the charged objects.

(c)On On the following diagram of object 1, draw and vectors label vectors to represent the forces (c) the following diagram of object 1, draw and label to represent the forces on the on the object. object.
(d) Using the conditions of equilibrium, write but do not solvetwo equations that could, together, be solved (d) Using the conditions of equilibrium, write but do not solvetwo equations that could, together, be solved for ! and the tension T in the left-hand string. for ! and the tension T in the left-hand string.

d) Using the conditions of equilibrium, writebut do not solvetwo equations that could, together, be conditions solved forof qequilibrium, and the tension T in the string.equations that could, together, be solved (d) Using the write but doleft-hand not solvetwo
for ! and the tension T in the left-hand string.

2.3.

2009 The College Board. All rights reserved. College 2009 The College Board. All rights reserved. Visit the Board on the Web: www.collegeboard.com. Visit the College Board on the Web: www.collegeboard.com.

-7-7-

GO ON TO THE NEXT PAGE. GO ON TO THE NEXT PAGE.

15

IC. AP Problems with Kinematic Applications 1. A particle of charge Q and mass m is accelerated from rest through a potential difference V, he magnitude of theattaining electric field at the originK. O? a kinetic energy What is the kinetic energy of a particle of charge 2Q and mass m/2 kQ kQ potential difference? kQ 2 kQ that from rest throughe. the2same b. c. is 2accelerated d. 2 2 2 (B) 2 (C) (D) 2K (E) 4K d dK d (A) 4 2d

b.

c.

d.

e.

2. An electron e and a proton p are simultaneously released from rest in a uniform electric field E, as shown above. Assume that the particles are sufficiently far apart so that the only force acting on each particle after it is released is that due to the electric field. At a later time when the particles are still in the field, the electron and the proton will in have the same on e and a proton p are simultaneously released from rest a uniform electric (A) direction of motion (B) speed displacement own above. Assume that the particles are sufficiently far apart so(C) that the only (D) magnitude ofthat acceleration (E) magnitude of force on each particle after it is released is due to the electric field. At a later time acting on them ticles are still in the field, the electron and the proton will have the same of motion b. speed 3. An electron is accelerated from rest for a time of 10-9 second by a uniform electric field that ent d. magnitude of-15 acceleration exerts a force of 8.0 x 10 newton on the electron. What is the magnitude of the electric field? e of force acting on them (A) 8.0 x 10-24N/C (B) 9.1 x 10-22 N/C (C) 8.0 x 10-6N/C (D) 2.0 x 10-5 N/C
4

c. 0.100 m d. 0.101 m e. 0.110 4. For the preceding problem, the speed of the m electron after it has accelerated for the 10-9 second is most nearly 1 the following will occur if(A) the 10 average of 3 the gas molecules in a closed m/s speed (B) 10 m/s (C) 105 m/s (D) 107 m/s (E) 109 m/s er is increased? y of the gas will decrease. b. is The density of 5. An electron volt a measure of the gas will increase. ure of the gas will increase. d. The pressure gas will decrease. (A) energy (B) electric field of the (C) electric potential due to one electron erature of the gas will decrease. (D) force per unit electron charge (E) electric charge b. 0.099 m

(E) 5.0 10 N/C late with sides of length 1.00 mx has a hole in its center 0.100 m in diameter. If the entire plate is temperature that its sides become 1.01 m long, the diameter of the hole will be

rical mirror shown above has a center of curvature at point c. Which point is focal point? b c. c d. d e. e

uencies of the first two overtones (second and third harmonics) of a vibrating nd 3f /2. What is the fundamental frequency of this string? b. f /2 c. f d. 2f e. 3f

t is placed in front of a converging thin lens at a distance from the center of the half the focal length. Compared to the object, the image is d larger b. upright and smaller c. inverted and larger nd smaller e. inverted and the same size

Copyright 2004 by College Entrance Examination Board. All rights reserved. central.collegeboard.com (for AP professionals) and www.collegeboard.com/apstudents (for AP students and parents)

2.3.

16

The Electric Potential Between Two Plates The electric field in the space above and below a charged sheet does not change with distance from the sheet so the work required to move a charge between the plates is simply the force on the moving charge multiplied by the distance between the plates. That is the potential energy change when the charge is transferred. And the electric potential in the space between the plates is this potential energy change divided by the magnitude of the charge.
V= !U W Fd qEd " d = = = = q q q q #

where ! is the surface charge density on each plate. Q/A, and d is the distance between the plates. A pair of charged plates is an essential circuit element known as a capacitor. In a circuit it functions as a charge storage device. But it is much more than a battery. A capacitor can be used to control the time dependence of a circuit. And unlike the battery the capacitor can discharge very rapidly to produce short-time events with very high power, such as a laser. The capacitors in a circuit are typically cylindrical. The pair of planes has been covered by an insulator and then rolled into a cylinder. The space between the parallel plates is usually filled with a material with a high permittivity, $. As $ increases the amount of charge that can be stored on the plates increases at fixed voltage. Walk across the rug with your wool socks on a crisp, dry winter day and the charge on your fingertip can leak painfully off to the doorknob. Why is a crisp, dry day the prime time for this effect? It is because water vapor in the air reduces capacitance, C.

Q = CV As charge is transferred from the socks to the rug the electric potential increases. The rate at which the charge increases as the potential increases is called the capacitance, C. The capacitance is affected by the material in the space between the charged surfaces.
Capacitance is also a property of the geometry of the charged surfaces. A large plate can hold more charge. Closely spaced plates can hold more charge.
Q = CV = CQd " C = !A / d A! parallel plate

(2.7)

The capacitance has units of Coulombs per Volt. And that is called a farad. The farad is much larger than common capacitors provide. The microfarad, mF, is often used. Since the work done in adding charges Q and Q to the plates increases with amount of charge already on the plates. This is analogous to the work done to stretch a spring. Like the spring the energy is equal to QV/2=Q2/2C.

2.3.

17

IC. AP Capacitor Problems 1. A 4 F capacitor is charged to a potential difference of 100 V. The electrical energy stored in the capacitor is (A) 0.2 nJ (B) 20 nJ (C) 2 J (D) 0.2 mJ (E) 20 mJ 2. Two parallel conducting plates, separated by a distance d, are connected to a battery of emf E. Which of the following is correct if the plate separation is doubled while the battery remains connected? (A) The electric charge on the plates is doubled. (B) The electric charge on the plates is halved. (C) The difference between the plates is doubled. 13. Which of potential the following will cause the electrical resistance of certain materials known as superconductors to suddenly decrease to essentially zero? (D) The potential difference between the plates is halved. (A) Increasing the voltage applied to the material beyond a certain threshold voltage (B) Increasing the pressure applied to the material beyond a certain threshold pressure (E) The capacitance is unchanged. to a constant voltage source. The magnitude of the electric field between the plates is of 2,000 N/C. If the voltage is doubled and the distance 14. Kirchhoffs loop rule for circuit analysis is an expression which of the following? (A) Conservation of chargeis reduced (B) Conservation of energy (C) Ampere's law between the plates to 1/5 the original distance, the magnitude of the new electric field (D) Faraday's law (E) Ohm's law is Questions(A) 15-16 800 refer to the circuit shown below. N/C N/C (B) 1,600 (C) 2,400 N/C (D) 5,000 N/C (E) 20,000 N/C 4. A parallel-plate capacitor has a capacitance C0. A second parallel-plate capacitor has plates with twice the area and twice the separation. The capacitance of the second capacitor is most nearly (A) C0./4 (B) C0/2 (C) C0 (D) 2C0. (E) 4C0 5. The charge stored in the 5-microfarad capacitor is most nearly (D) 1,100 C (E) 1,800 C
(C) Cooling the material below a certain threshold temperature (D) Stretching the material to a wire of sufficiently small diameter Placing the material in a sufficiently plates large magnetic 3. (E) Two parallel conducting are field connected

15. The equivalent capacitance for this network is most nearly (A) 10/7 F (B) 3/2 F (C) 7/3 F (D) 7 F (E) 14 F

16. The charge stored in the 5-microfarad capacitor is most nearly (A) 360 (B) 710 C (A) 360 C (B) 500 C C (C) 710 C 500 (D)C 1,100 C (C) (E) 1,800 C

17. Two large parallel conducting plates P and Q are connected to a battery of emf !, as shown above. A test charge is placed successively at points I, II, and III. If edge effects are negligible, the force on the charge (A) ofitequal magnitude and in the same direction as the force on the charge when it is at point when is at point III is (A) of equal magnitude and in the same direction as the force on the charge when it is at point I (B) of equal magnitude and in the same direction as the force on the charge when it is at point (B) of equal magnitude and in the same direction as the force on the charge when it is at point II (C) equal in in magnitude to the force on the the charge when it is at point I, but in when the opposite direction (C) equal magnitude to force on the charge it is at point I, but in the opposite (D) much greater in magnitude than the force on the charge when it is at point II, but in the same direction direction (E) much less in magnitude than the force on the charge when it is at point II, but in the same direction

6. Two large parallel conducting plates P and Q are connected to a battery of emf as shown at the right. A test charge is placed successively at points I, II, and III. If edge effects are negligible, the force on the charge when it is at point III is I II

(D) much greater in magnitude than the force on the charge when it is at point II, but in the same direction (E) much less in magnitude than the force on the charge when it is at point II, but in the same direction

18. The direction of the magnetic field at point R caused by the current I in the wire shown above is (A) to the left (B) to the right (C) toward the wire (D) into the page (E) out of the page

Copyright 1993 by College Entrance Examination Board. All rights reserved. College Board, Advanced Placement Program, AP, SAT, and the acorn logo are registered trademarks of the College Entrance Examination Board.

2.3.

18

Questions 45-46

Two large, flat, parallel, conducting plates are 0.04 m apart, as shown at the right. The lower plate is at a potential of 2 V with respect to ground. The upper plate is at a potential of 10 V with respect to ground. Point P is located 0.01 m above the lower plate. 7. The electric potential at point P is
Two large, flat, parallel, conducting plates are 0.04 m apart, as shown above. to ground. Point P is located 0.01 m above the lower plate.

(A) 10 (B) 8 of V2 V (C) 6V (D) V (E) 2V at V a potential with respect to4 ground. The upper plate is at a potential o

8. The magnitude of the electric field at point P is 45. The electric at point is V/m (A) 800 V/m (B) 600 V/m (C) 400 V/m (D) potential 200 V/m (E)P 100
a. 10 V b. 8 V c. 6 V 46.

d. 4 V

e. 2 V d. 200 V/m e.

The magnitude of the electric field at point P is a. 800 V/m b. 600 V/m c. 400 V/m

47.

Two conducting wire loops move near a very long, straight conducting current I. When the loops are in the positions shown above, they are movin shown with the same constant speed v . Assume that the loops are far enou do not affect each other. Which of the following is true about the induced e any, in the loops? a. b. c. d. e. Loop l No current No current Clockwise direction Clockwise direction Counterclockwise direction Loop 2 No current Counterclockwise direction No current Clockwise direction Clockwise direction

Copyright 2004 by College Entrance Examination Board. All rights reserved. Visit apcentral.collegeboard.com (for AP professionals) and www.collegeboard.com/apstudents (for AP students a

2.3.

19

i1

Circuits with Capacitors Batteries are designed to provide a sustained release of electrical energy. Often we want the energy transport to occur at a very high i2 rate. Good examples are the flash bulb, the paddles used to restart the heart in a defibrillator, or a laser.

When a voltage is applied through an external circuit connecting a pair of metal plates separated by an insulating gap, charges move to the plate of the capacitor that is physically connected to the positive pole of the battery. The same amount of charge is removed from the other plate. For this simple circuit with an EMF E over the battery, energy conservation gives E q/C=0.

i1 i2 i3

With two capacitors in a circuit that are connected in parallel, energy conservation is applied over both loops

i2 loop : E -

q2 =0 C2

and

i3 loop : E -

q3 =0 C3

q1 = q 2 + q 3 E Ceq = E C1 + E C2 ! Ceq = C1 + C2

With two capacitor in a circuit that series that are connected in series, energy conservation is applied

!1 !1 q q 1 $ 1 $ E - 3 - 2 = E - q 2 # + & = 0 Ceq E = q ' Ceq = # + & C3 C 2 " C3 C 2 % " C3 C 2 %

(1

2.3.

20

AP Problems Involving Circuits with Capacitors


a
C1 C1 C3 C 3 C2 C2 C 3

61. CA push of mass m is pushed rough horizontal floor 1.broom Rank the equivalent capacitances ofacross the four a circuits, greatest C 1 2 first. b shown above. The coefficient of friction between the broom an as broom (A) (mg + Tsin!) (B) (mg-Tsin! d C2 has magnitude C1 C (D) (mg-Tcos!) (E) mg 3

2. Two capacitors are connected in parallel as shown at the right. A voltage V is applied to the pair. What is the ratio of charge stored on C1 to the charge stored on C2, when C1=1.5 C2. Using equation editor show your work.

62. Two are connected parallel asthe shown above. A volta 3.capacitors Find the equivalent capacitance in of the circuit at left. Put C1 = of charge stored charge stored on C2, to when 1 to 10.0 F, C2 = 5.0on F,C and C3the = 4.0 F. Use equation editor show C1 = 1.5C (A) your 4/9 work. (B) 2/3 (C) 1 (D) 3/2 (E) 9/4

4. If the dielectric material between the plates of a capacitor begins to leak charge (breaks down) then the capacitor begins to act like a bare wire. Suppose that capacitor 3 breaks down, becoming equivalent to a conducting path. What changes in (a) the charge and (b) the potential difference occur for capacitor 1? Assume that V=100 V. Use equation editor to show your work.

63. Two long, parallel wires, fixed in space, carry currents I1 and I2. currents will give an attractive force of magnitude 4F? (A) 2I1 and !I2 ( B) I1 and "I2 (C) !I1 and !I2 (D) 2I

(c) Find the charge, potential, and stored energy for each capacitor.

64. The circuit shown above left is made up of a variable resistor and graph of the power P dissipated in the resistor as a function of the above right. What is the emf of the battery? (A) 0.025 V (B) 0.67 V (C) 2.5 V (D) 6.25 V (E) 40 V

2.3.

65. A body moving in the positive x direction passes the origin at time body has a constant speed of 24 meters per second. At t = 1 secon 6 meters per second squared in the negative x direction. The posit (A) +99m (B) +36m (C) - 36 m (D) - 75 m 21 (E) - 9

66. A person weighing 800 newtons on Earth travels to another planet

5. The EMF, E, in the circuit at the left is 12 V. Find the charge on each capacitor (a) when initially on switch S1 is closed and (b) later when switch S2 is also closed. Put C1 = 1.0 F, C2 = 2.0 F, C3 = 3.0 F, and C4 = 4.0 F. Use equation editor to show your work.

6. The total capacitance of several capacitors in parallel is the sum of the individual capacitances for which of the following reasons? (A) The charge on each capacitor depends on its capacitance, but the potential difference across each is the same. (B) The charge is the same on each capacitor, but the potential difference across each capacitor depends on its capacitance. (C) Equivalent capacitance is always greater than the largest capacitance. (D) Capacitors in a circuit always combine like resistors in series. (E) The parallel combination increases the effective separation of the plates. 7. The resistances in schematics a and b below are all 6.0 % and the batteries are ideal 12 V batteries. (a) When the switch in circuit a is closed, what is the change in the electric potential across resistor 1, or does this potential remain unchanged? (b) When the switch is closed in circuit b, what is the change in the electric potential across resistor 1, or does this potential remain unchanged? Use equation editor to show your work.

8. Rank the equivalent resistances of the four circuits at the left, greatest first.

a
R 1

R1 R3 R 3

R 2

R 3

R2

b d

R 1

R 2

R1

R3

R2

2.3.

22

9. If the potential at point P1 is 100 V, what is the potential at point P2? Describe your solution in words.

10. Find the current in each resistor and the potential difference between points P1 and P2. Put E1 = 6.0 V, E2 =5.0 V, E3 = 4.0 V, R1 = 100 %, and R2 = 50 %. Use equation editor to show your work, but work it out first on a piece of paper.

11. Find the current at P for when all switches are closed. Put E = 120 V, R1 = 20 %, and R2 = 10.0 %. Assume that the battery has no resistance. Describe your solution strategies and reasoning in words. Closing which single switch gives the largest current at P?

Closing which single switch gives the smallest current at P?

12. The graph at the left shows how electric potential varies through the circuit below it. If the battery is ideal with an EMF of 12 volts, what are the resistances R1, R2, and R3? What is the rate of energy dissipation through resistors R1, R2, and R3? Use equation editor to show your work.

2.3.

23

RC Circuits When a capacitor is discharged through a resistor, as it discharges the electric potential of the capacitor becomes smaller. But the rate at which the capacitor loses charge is the current in the resistor. Combining these together we get a time-dependent circuit called an RC circuit.

This equation also shows that the product RC has units of time. By choosing combinations of resistors and capacitors the characteristic time of the circuit can be chosen. This equation is a differential equation. In AP-B Physics you do not have to solve the differential equation. You only need to know what the solution is. There are actually two solutions. One describes the process of charging the capacitor and the other describes the discharge of the capacitor. The switch in the circuit can be used to change the circuit from one that charges the capacitor to a circuit that discharges the capacitor. The solutions are charging: Q = CE (1 ! e ! t/RC ) (2.8)

discharging: Q = CEe ! t/RC (2.9)


Examples 1. A capacitor with initial charge q0 is discharged through a resistor. What multiple of the time constant & = RC gives the time the capacitor takes to lose (a) the first one-third of its charge and (b) two-thirds of its charge? (2.10) Using your calculator to evaluate the natural log of 1/3 and 2/3 the ratios t/& are 1.1 and 0.41, respectively. 2. What multiple of the time constant & gives the time taken by an initially uncharged capacitor in an RC circuit to be charged to 99.0% of its final charge. (1 ! e ! t/RC ) = 0.99 " e ! t/RC = 0.01 " t = !# ln(0.01) = 4.6# This is a rule of thumb that engineers use when dealing with potentially charged capacitors. A capacitor sitting in a circuit may have been disconnected from a circuit before it was discharged. A capacitor can store a lot of charge and if it discharges into your fingers you can get a shock. So the engineer lays a wire over the leads of the capacitor for 5 time constants to avoid the shock.
# Q& Q Q = = e ! t/RC " ! ln % ( = t / ) CE Q0 $ Q0 '

2.3.

24

SL Measuring the Time Constant in a Charging RC Circuit Build the circuit at the left using: Breadboard Capacitor Resistors Sparkvue voltmeter About breadboards There is continuity along the numbered rows only Short pieces of wire can be used as jumpers About capacitors Some capacitors have a polarity (-ve sign in a battery symbol usually) Capacitors can be fully discharged by shorting across the leads for 4-5 RC As a quick guide, after 1 RC the voltage should be at V0(1-e-1) = 0.63 V0 Values of capacitance have uncertainties of 30% About Resistors 3 or 4 colored bands (rightmost is precision gold, 5%, or silver, 10%) From the left read one or two digits, then power of ten: Black(0), Brown(1), Red(2), Orange(3), Yellow(4), Green(5), Blue(6), Violet(7), Grey(8), White(9). e.g. OOR is 3300 ohms. About voltmeters Voltmeters have large but finite resistance. If you attach the voltmeter over the capacitor the capacitor will discharge through the voltmeter (not what you wanted to happen). The sampling frequency needs to be much greater than 1/RC (at least 50 Hz) About LEDs LEDs are biased. Current only flows when a threshold voltage is present and current only flows in one direction (from long to short lead). About your materials Your group has been given 1 capacitor, 5 resistors, 2 LEDs and a battery At the end of the class period return these to the labeled baggie Using your circuit Fully discharge the capacitor with the battery disconnected Start collecting data (green arrow) and display on a voltage vs time graph Connect the battery Stop collecting data when current ceases Repeat for each combination of resistor and capacitor Save the data to a flash drive (connection icon -> save as file) The txt files can be imported to spreadsheet and analyzed to determine RC Report Each assigned lab group produces one report Format: 1 paragraph summary of RC circuit principles (20%), V(t) graphs for each R (20%), table with three columns (R, measured RC, and theoretical RC) (30%), and 1 paragraph summary (10%) Demonstrated the charge-discharge circuit (20%) 2.3.
25

9-volt battery

IC. AP Problems on RC Circuits 1. The switch in the circuit at the left is closed at time, t = 0, to begin charging an initially uncharged capacitor of capacitance 15.0 F through a resistor with a resistance of 20.0 %. At what time is the potential across the capacitor equal to that across the resistor? Show your work with equation editor.

2. In the circuit above the EMF is 12.0 V, the resistance is 4.0 %, and the capacitance is 4.0 F. The capacitor is initially discharged and the switch is closed. Then what is the current through the resistor when the charge on the capacitor is 8.0 C. Show your work with equation editor.

2.3.

26

2007 AP PHYSICS B FREE-RESPONSE QUESTIONS (Form B)

ts)

3, In the circuit at the left, a 12.0 V battery is connected to two resistors, one of resistance 1000 % and the other of resistance 500 %. A capacitor with a capacitance of 30 !10-6 F is connected in parallel with the 500 % resistor. The circuit has been connected for a long time, and all currents have reached their steady states. (a) Calculate the current in the 500 % resistor. Show your work with equation editor.

rcuit above, a 12.0 V battery is connected to two resistors, one of resistance 1000 ! and the other of

ce 500 ! . A capacitor with a capacitance of 30 10 6 F is connected in parallel with the 500 ! The circuit has been connected for a long time, and all currents have reached their steady states.

culate the current in the 500 ! resistor.

(b) i. Using the Shapes tool draw an ammeter in the circuit above in a location such that it could Draw an ammeter in the circuit above in a location such that it could measure the current in the 500 ! measure the current in the 500 % resistor. Use the symbol to indicate the ammeter.
resistor. Use the symbol to indicate the ammeter.

ii. Draw a voltmeter in the circuit above in a location such that it could measure the voltage across the 1000 Use the symbol to indicate the voltmeter. Draw a voltmeter in the circuit above% in resistor. a location such that it could measure the voltage across the
1000 ! resistor. Use the symbol

(c) Calculate the charge stored on the capacitor. Show your work with equation editor.

to indicate the voltmeter.

culate the charge stored on the capacitor.

culate the power dissipated in the 1000 ! resistor.

(d) Calculate power dissipated in the 1000 % resistor. Showof your work with equation editor. 500 ! resistor is removed and replaced by a resistor greater capacitor is now discharged, and thethe stance. The circuit is reconnected, and currents are again allowed to come to their steady-state values. Is charge now stored on the capacitor larger, smaller, or the same as it was in part (c)?
____ Smaller

_ Larger

ify your answer.

(e) The capacitor is now discharged, and the 500 % resistor is removed and replaced by a resistor of greater resistance. The circuit is reconnected, and currents are again allowed to come to their steady-state values. Is the charge now stored on the capacitor larger, smaller, or the same as it was in part (c)? ____ Larger ____ Smaller ____ The same as Justify your answer.

____ The same as

2007 The College Board. All rights reserved. apcentral.collegeboard.com (for AP professionals) and www.collegeboard.com/apstudents (for students and parents).

2.3.
-7-

GO ON TO THE NEXT PAGE.

27

2007 AP PHYSICS B FREE-RESPONSE QUESTIONS

4. The circuit above contains a battery with negligible internal resistance, a closed switch S, and 3. (15 points) three resistors, with a resistance of R or 2R . The circuit above each contains a battery with negligible internal resistance, a closed switch S, and three resistors, (a) with a resistance of R or 2R. each
(a)

i. Rank the currents in the three resistors from greatest to least, with number 1 being greatest. If two resistors have the same current, give them the same ranking.
____ IA ____ IB ____ IC

i. Rank the currents in the three resistors from greatest to least, with number 1 being greatest. If two resistors have the same current, give them the same ranking. I I I A B C

ii. Justify your answers.


(b) (b)

ii. Justify your answers.

i. Rank the the voltages across the three greatestto toleast, least, with number 1 being i. Rank voltages across the threeresistors resistors from from greatest with number 1 being greatest. If two greatest. If two resistors have theacross same them, voltage across give them the same ranking. resistors have the same voltage give them them, the same ranking.
ii. Justify your answers. A B C V A VB VC ____ V ____ V ____ V

For parts (c) through (e), use ! = 12 V and!R = 200 " . (c) Calculate equivalent resistance ofV the circuit. For parts (c)the through (e), use E = 12 and R = 200 %. (d) Calculate the current in resistor RC . (e) The switch S is opened, resistor RB is removed and replaced by a capacitor of capacitance 2.0 10 6 F , and the switch S is again closed. Calculate the charge on the capacitor after all the currents have reached their final steady-state values.

ii. Justify your answers.

(c) Calculate the equivalent resistance of the circuit.

(d) Calculate the current in resistor RC. (e) The switch S is opened, resistor RB s removed and replaced by a capacitor of capacitance 2.0 !10-6 F, and the switch S is again closed. Calculate the charge on the capacitor after all the currents have reached their final steady-state values.

2007 The College Board. All rights reserved. Visit apcentral.collegeboard.com (for AP professionals) and www.collegeboard.com/apstudents (for students and parents).

2.3.
-8-

GO ON TO THE NEXT PAGE.

28

Storage Devices Consider these case posts: Lithium-ion battery Electrical double layer capacitors Graphene supercapacitors Case Rubric Opportunities to construct a summary of an Engineering Case will be provided throughout the year. Each student must post two cases. Your summaries are no more than 5 MByte. They should be posted on techphysics.org. The summary should include: Statement defining the problem addressed by the design (20 pts) Statement defining the essential features of the design (20 pts) If the design or technology failed, what causes were identified? (30 pts) or If the design or technology succeeded, what were the consequences? (30 pts) or If sequence is considered, how has the design or technology evolved? (30 pts) Sources used in your review are cited and credible (10 pts) The work is free of grammatical and spelling errors (20 pts) Each student must review five posts. The review is posted at the site with the document reviewed. Your review is 300 words or less. The reviews should include: Identify author and title of work reviewed (5 pts) Criticism is constructive and well-reasoned (20 pts)

2.3.

29

Ferromagnetic Symmetry Many naturally occurring rocks are ferromagnetic. These ores can be used to produce permanent magnets. Youve used a compass so you know the phenomenon. To accept that something happens and use that knowledge without understanding what causes it to happen is called phenomenological understanding. Engineers have to do it all of the time since the systems with which they work are usually too complex for complete understanding. An early engineer might have been confronted with this situation. There are four bars. It is observed that a and d attract, c and f repel, e and h attract, and a and h attract. Our engineer, as all engineers must, made a decision and called b south. Which ends are north? Are all of the bars magnets?

a c

e g

b Sd f h

If a magnet is cut into two pieces both pieces have the same symmetry. The conclusion is that a magnetic monopole can never be isolated or found alone in nature. There was to explain of this phenomenon until quantum mechanics was developed. P.A.M. Dirac, who predicted the existence of antimatter long before it was observed also predicted that magnetic monopoles should exist. We keep looking. However, the field lines were well known to early navigators. The compass needle is a magnet and will align with the magnetic field lines of the Earth. These lines are loops that connect the geomagnetic south pole (north seeking and presently in the Artic) with the geomagnetic north pole (south seeking and presently in the Antarctic). We do not know as yet the mechanism that creates the Earths magnetic field. But we know that the orientation of the Earths magnetic dipole undergoes large oscillations and even flips over. We are due for a flip. The field strength is currently decaying. Do a google and youll get the decay rate. One of the ideas that will emerge here is that the magnetic bottle; a magnetic field can confine charged particles. The Earths magnetic field causes the Earth to retain an atmosphere. Mars has an atmosphere that is roughly 1000 times less dense than that of Earth. Mars has no magnetic field. You observe the interaction of the Earths magnetic field with the charged particles of the upper atmosphere when you observe the aurora borealis at night in the northern hemisphere. The SI unit of the magnetic field strength is the Tesla, T. 1 T = 1 Ns/Cm.

2.3.

30

Lorenz Force A current flowing through a wire deflects a compass needle. So an interpretation is that moving charge produces a magnetic force, producing a torque that acts on the permanent magnet of the compass needle. Newton's 3rd law tells us that forces act in pairs. If the current produces a force that acts on the magnet, then the magnet produces a force that acts on the charges in the current. There must be a force acting on a charged particle moving through a magnetic field. Forces are measured by measuring the effects that they produce. A charged particle is injected into a region in which there is a magnetic field. The empirical relationship between the force, the velocity of a moving charge, v, and the magnetic field, B, is found This is called the Lorentz force.

F = qv ! B

Alert! Alert! The direction of the force is the most important result here. The force is acting in a direction that is perpendicular to the plane formed by qv and B. Alert! Alert! See the q. If q is negative then qv is rotated by !. This means that the force vector is pointing the opposite direction to that constructed with the right-hand-rule (RHR). Practice the rule. Put your index finger along qv and your second finger along B. Your thumb points along the force vector. Heres a test.

+ve q F v B

-ve q v B F

v1 v3

v2 v4 B

Negatively charged particles with the velocities shown are moving through a uniform magnetic field with the orientation shown. Describe the directions of the forces on each particle. Remember, it is vxB! The v is first and order matters.

Using the RHR for a positive particle I get forces for 1 and 2 that point into the page and forces for 3 and 4 that point out of the page. The problem involves a negatively charged particle so these directions are reversed. To calculate cross product you can use this
i ! ! A ! B = Ax Bx j Ay By k Az = i BZ Ay By Az BZ " j Ax Bx Az BZ +k Ax Bx Ay By A B "A B = i A yBZ " A zBy " j ( A xBZ " A zB x ) + k x y y x

(2.11)

2.3.

31

IC. Lorenz Force Problems 1. For four situations, the velocity v of a proton at a certain instant as it moves through a uniform magnetic field B are given. Rank the situations according to the magnitude of the magnetic force on the proton, greatest first. Show your work by expressing the i, j, k components using equation (2.11) on the preceding page.

2. Use the Lorenz force to show that the definition of a Tesla is dimensionally consistent. Show your work with equation editor. 3. Each of the electrons in the beam of a television tube has a kinetic energy of 12.0 keV. What is the velocity of the electron? Recall 1eV=1.610-19 J and me = 910-31 kg. Show your work with equation editor.

The tube is oriented so that the electrons move horizontally from geomagnetic south (geographic north) to geomagnetic north (geographic south). The vertical component of Earths magnetic field points down within the TV tube and has a magnitude of 55.0mT. What is the magnitude of the acceleration? Recall FB=qvB and 1 T (tesla) is 1 Ns/Cm. Show your work with equation editor.

In what direction will the beam deflect? Hint: The central concept that needs to be digested very well is the relative directions of qv, B, and F. It so important that it needs to be repeated. The force is perpendicular to the plane formed by qv and B. Perpendicular could mean up or down relative to that plane. And there is a charge present, so you know that there is a sign convention. The choice made for the sign in the empirical equation says that if the particle is positively charged then F is up out of a plane defined in a right-handed frame as v is rotated into B.

2.3.

32

Crossed Fields This is the idea that brought you TV. But it also brought the charge and mass of electron (as the ratio only) and the first "elementary particle." A positively charged particle in an electric field experiences a force which is parallel to the direction of the electric field, F=qE. This force accelerates the charged particle in the direction of the field. Taking the electric field direction along the y-axis the electron feels a force FE,y = -|qE|j due to F this field. A magnetic field perpendicular to the electric field will also produce a E F force on this charge in motion. The direction of the force on a charged particle due B to a magnetic field is perpendicular to the plane formed by the velocity vector and the magnetic field vector. We take the z axis as the direction of the magnetic field, then the force also lies along the y axis, FB, = |qvB|j To get the proper direction of the magnetic field force you have to use your right hand to visualize. The magnetic force as pointed toward your nose as you gaze into the page along the -k direction toward the x-y plane on which v is directed to the right. Your thumb points down. But the electron is negatively charged so the direction of the thumb is flipped and the magnetic field is up.

E v

+ve q F v B

-ve q v B F

A convention used by the AP-B exam is to display the direction of fields that are perpendicular to the plane of the page with and x. If the field is directed out the page towards you then it is denoted with a filled circle. If the field points into the page it is denoted with an x.

Since the magnetic and electric fields are perpendicular, the forces are anti-parallel, and the effects of the electric field and the magnetic field on a charged eE 2 FE = eE ! "y = t particle can be isolated.
2me

This was the objective in J.J. Thomson's design of the cathode ray tube. To express the deflection 'y without needing to measure the very small time of flight of an electron we apply Newton's 2nd to an electron accelerated by an electric field. We shoot an electron toward a screen a distance L away along the x-axis. The electric potential drop along the y-axis produces a force. The electron is accelerated towards the positively charged plate that generates this potential drop.
x = ve,x t ! y = eEx2 eEL2 ! " y(L) = 2mev 2 2me v2 e,x e,x

There is no field in the x direction. So the velocity along x is constant. This defines the time of flight. Measurement of the y displacement can be made. E and L are known.

2.3.

33

The velocity is inconvenient and needs to be eliminated. That is accomplished by the crossed magnetic field.
eve,xB = eE ! v e,x = The force due to the magnetic field is multiplied by the charge on B the electron. So the magnetic force acting on the electron will be directed along the negative y-axis. It opposes the force due to the electric field. E

The magnitudes of the electric and magnetic field can now be tuned and the ratio of the mass and charge of the electron can be determined. Thats Nobel stuff.

!y(L) =

eEL2B2 m L2B2 " e = 2 e 2!yE 2meE

2.3.

34

Crossed Field Example


3, 4, 5 & 6

xE

1&2

7&8 9 & 10 The electric force will either be out of the screen (negatively charged particle) or into the screen (positive charged particle). Confirm that for yourself.

Shown here are electric and magnetic fields with perpendicular orientations and the velocities of several particles. With these field orientations the electric and magnetic forces may act in opposite directions.

The magnetic force for velocity vectors 1 through 6 may act either into or out of the plane of the screen. Confirm that for yourself and define the direction for positively and negatively charged particles. If the electric and magnetic forces are oppositely directed and balanced, the velocity is equal to the ratio E/B.

particle 1 2 3 4 5 6 7 8 9 10

charge + + + + + + -

speed less than greater than less than greater than less than greater than less than greater than less than greater than

The magnetic force is proportional to velocity and the electric force is independent of velocity. So as the velocity of the charged particle increases the magnetic force can dominate the electric force. As the velocity of the charged particle decreases the electric force can dominate the magnetic force. So positively charge particles will be directed into the plane by E and negatively charged particles will be directed out of the plane.
particle 1 2 3 4 5 6 7 8 9 10 direction into

In each case the magnetic field points to the left. So positive particles with a velocity upward will be deflected out of the plane and positive particles with a velocity downward will be deflected into the plane. Variation in the relative speed can only affect those cases in which the electric field and magnetic field produce accelerations that are in opposite directions.

These cases arise when the velocity is upward. Confirm that for yourself.

2.3.

35

Complete Cases particle 1 2 3 4 5 6 7 8 9 10 outward. For cases 5 & 6 the electric force is directed out of the screen and the magnetic force points in. If speed is low the deflection is out of the screen. If the speed is high the deflection is into the screen. direction into into into out of out of into into into out of out of So the simple cases are: 1 & 2 (both forces into), 7 & 8 (electric force into and magnetic force vanishes), and 9 & 10 (electric force out of and magnetic force vanishes). Case 3 has a low speed and this makes the electric force dominant; the deflection is into the screen. Case 4 has a dominant magnetic force and the deflection is

2.3.

36

Example Calculations 1. A proton travels through uniform magnetic and electric fields. The magnetic field is 2.5 i mT. At one instant the velocity of the proton is 2000 j m/s. At that instant, what is the magnitude of the net force acting on the proton if the electric field is a) 5.0 k V/m, b) -5.0 k V/m, and c) 5.0 i V/m? Since the particle is positive the magnetic force points along k in all cases. The magnitude is 5e N, where e is the fundamental charge. a) Both forces have the same direction. b) The magnitude is 0; the forces cancel. c) The magnitude is the square root of the sum of squares. 2. An electron is accelerated through a potential difference of 1.0 kV and directed into a region between two parallel plates separated by 20 mm with a potential difference of 100 V. The electron is moving perpendicular to the electric field when it enters the region between the plates. What magnetic field is necessary perpendicular to both the electron path and the electric field so that the electron travels in a straight line? The electron is moving perpendicular to the electric field when it enters the region between the plates. The force due to the electric field must balance the magnetic force. eE=qvB. And applying energy conservation to the electron the velocity can be determined mv2/2=eV.
B= E 100V / 20mm = = v 2(1kV)(e / m) 5kV / m 2(1000J / C)(1.6x10!19 C / 9.11x10!31kg) = 0.27mT

2.3.

37

68. A constant force of 900 N pushes a 100 kg mass up the inclined plane shown above at a uniform speed of 4 m/s. The power developed by the 900 N force is most nearly a. 400 W b. 800 W c. 900 W d. 1000 W e. 3600 W AP B Field Questions

1. As shown above, a positively charged particle moves to the right without deflection through a 69. of charged As shown above, a positively charged particle moves to the right without deflection pair plates. Between the plates are a uniform electric field E of magnitude 6.0 N/C through a pair of charged plates. Between the plates are a uniform electric field of speed of and a uniform magnetic field B of magnitude 2.0 T, directed as shown in the figure.E The magnitude 6.0 N/C and a uniform magnetic field B of magnitude 2.0 T, directed as shown in the particle is most nearly the figure. The speed is most nearly (A) 0.33 m/sof the (B) particle 0.66 m/s (C) 3.0 m/s (D) 12 m/s (E) 18 m/s a. 0.33 m/s b. 0.66 m/s c. 3.0 m/s d. 12 m/s e. 18 m/s In the following show your work using equation editor. 70. A hollow metal sphere 1.0 m in diameter carries a charge of 4.0 C. The electric field at a distance the of 2.0 m from the center of the sphere is most nearly 2. Calculate in an electric field of 1.5 kV/m 5 3 minimum velocity of 4 an electron at equilibrium a. 9.0 x 10 N/C b. 1.8 x 10 N/C c. 2.4 x 104 N/C d. 3.6 x 104 N/C e. 1.4 x 10 and a magnetic field of 0.4 T. N/C

3. An electron is accelerated from rest through potential difference V1 = 1.00 kV enters the gap between two parallel plates separated by 20 mm. The potential of the upper plate relative to the lower plate is 100V. Neglect fringing and assume that the electrons velocity vector is perpendicular to the electric field vector between the plates. In unit-vector notation, what uniform magnetic field allows the electron to travel in a straight line in the gap?

4. A beam of 6Li ions is produced by a source. The ions have a charge of +e and a mass of 9.99 10-27 kg. The ions are accelerated by a potential difference of 30 kV and pass horizontally into a region in which there is a uniform vertical magnetic field of magnitude B = 1.2 T. Calculate the strength of the smallest electric field, to be set up over the same region, that will allow the 6Li ions to pass through undeflected.

5. An electron is sent through a region with both electric and magnetic fields. The dependence of the j-component of the net force of the velocity is given by -210-19 N + 25 Ns/m v. Using a coordinate system in which the electric field is along j and the magnetic field has no icomponent, find the magnitude of E and the vector form of B.

Copyright 2004 by College Entrance Examination Board. All rights reserved. Visit apcentral.collegeboard.com (for AP professionals) and www.collegeboard.com/apstudents (for AP students and parents)

2.3.

38

2011 AP PHYSICS B FREE-RESPONSE QUESTIONS

6. In the experimental setup represented at theQUESTIONS left, a very 2011 AP PHYSICS B FREE-RESPONSE small plastic sphere of mass m with charge q is allowed to fall under the influence of gravity between two parallel metal plates separated by a fixed distance L. A variable potential difference may be applied between the two plates. The experiment is conducted inside a vacuum chamber.
5. (15 points)

In the experimental setup represented above, a very bottom small plasticplates sphere ofsuch mass mthat with charge q is allowe (a) A potential difference of magnitude V is m applied between the top and mental setup represented above, a very small plastic sphere of mass with charge q is allowed to metal plates fall under the influence of gravity between two parallel separated by a fixed distance L . A variab the sphere falls at constant speed v. Derive an expression for the magnitude of the charge q on potential difference may be applied between the two plates. The experiment is conducted inside a vacuum e influence of gravity between two parallel metal plates separated by a fixed distance L.A variable chamber. the sphere.the Express yourThe answer in terms of m, L, V, and fundamental constants, as appropriate. erence may be applied between two plates. experiment is conducted inside a vacuum (a) A potential difference of magnitude V is applied between the top and bottom plates such that the sphere Show your work using the equation editor. falls at constant speed !. Derive an expression for the magnitude of the charge q on the sphere. Express your answer in terms of m, L, V, and fundamental constants, as appropriate.

ial difference of magnitude V is applied between the top and bottom plates such that the sphere The experiment performed many times with spheres of identical known mass but different unknown charge q on is the sphere. Express onstant speed !. Derive an expression for the magnitude of the charge each time adjusting the potential difference V to the value needed so that the sphere falls at constant speed ! wer in terms of m, L, V, and fundamental constants, as appropriate. The magnitudes of the charges are calculated from the measured values of the potential difference. The data i
plotted below as a function of the magnitude of V.

ent is performed many times with spheres of identical known masswith but different unknown charges, The experiment is performed many times justing the potential difference V to the value needed so that the sphere spheres of identical known mass but different falls at constant speed !. des of the charges are calculated from theeach measured of the potential difference. The data is unknown charges, time values adjusting the w as a function of the potential magnitude of V . difference V to the value needed so that

the sphere falls at constant speed v. The magnitudes of the charges are calculated from the measured values of the potential difference. The data is plotted below as a function of the magnitude of V. (b) Provide a physical explanation for the gap observed in the data between potential differences of 1700 V
and 2800 V.

2011 AP PHYSICS B FREE-RESPONSE QUESTIONS

(b) Provide a physical explanation for the gap observed in the data between potential differences of 1700 V and 2800 V.

alue of L is 0.050 m, calculate the mass m of the spheres.

magnetic field of magnitude B, directed into the page, is now applied in the bottom half of the region a physical explanation for the gap observed in the data between potential differences of 1700 V e plates, as shown in the figure below. The experiment is repeated, with the potential difference 0ain V.so that the charged sphere falls with constant speed prior to entering the magnetic field.
2011 The College Board. Visit the College Board on the Web: www.collegeboard.org.

(c) If the value of L is 0.050 m, calculate the mass m of the spheres.

A uniform magnetic field of magnitude B, directed into the GO ON TO THE NEXT PAG page, is now applied in the bottom -10half of the region between the plates, as shown in the figure below. The experiment is repeated, with the potential difference adjusted again so that the charged sphere falls with constant speed prior to entering the magnetic field.

escribe the motion of the sphere as it travels through the magnetic field. escribe how the motion could be used to determine the sign of the charge.

an expression for the minimum value of B needed to prevent the sphere from reaching the bottom Express your answer in terms of m, q, !, L, and fundamental constants, as appropriate.
2011 The College Board. Visit the College Board on the Web: www.collegeboard.org.

2.3.

39

GO ON TO THE NEXT PAGE. -10-

(d) i. Describe the motion of the sphere as it travels through the magnetic field.

ii. Describe how the motion could be used to determine the sign of the charge.

(e) Derive an expression for the minimum value of B needed to prevent the sphere from reaching the bottom plate. Express your answer in terms of m, q, u, L, and fundamental constants, as appropriate. Use the equation editor to show your work.

2.3.

40

Charges Go in Circles in a Magnetic Field You are sitting in a magnetic field with a north pole on the ceiling and a south pole on the floor. An electron shoots out of your eye. Use your right hand to show that there is a force that acts from the right toward the left. The electron is continuously deflected toward your left. Follow the electron with your right hand and you'll see it moves in a circle around you. You can simply calculate the time required to make one complete pass. The magnitude of the force is just evB. m v2 2"me 2"me evB = e ! eB = !T= (2.12)
r T eB

The path is a uniform circular motion. The period is inversely proportional to the magnitude of the field; stronger fields produce faster cycling. This is the basic idea behind the particle accelerator where particles are accelerated to extremely high energies, TeV now, and collide. The structure of the nucleus is revealed by examining collision products. At Argonne, Livermore and Brookhaven National labs rings are used to accelerate electrons. An intense beam of x-rays is produced by these very high velocity electrons. The electron was launched from your eye with no velocity component along the magnetic field. Then the electron moved in a uniform circular motion in a plane parallel to the floor. If the electron is launched with a velocity component in the direction of the floor it will spiral downward. Solar radiation has a distribution of frequencies that includes x-rays. High frequency x-ray radiation is intercepted by Earth's upper atmosphere. The energy of an x-ray is sufficient to ionize nitrogen atoms. The result is a shell, called the ionosphere, of charged particles moving at very high velocities. These charges interact with the magnetic field of the Earth. They spiral "downward" along the magnetic field lines. These electrons can penetrate nearly to the stratopause. They excite electrons in atmospheric nitrogen and oxygen. These electrons relax by emitting radiant energy. We see the result as the aurora borealis, even on Saturn. Example calculations An electron is accelerated from rest by a potential difference of 350 V. It then enters a uniform magnetic field of magnitude 200 mT with its velocity perpendicular to the field. Calculate the speed of the electron and the radius of its path in the magnetic field. You know the constant acceleration so energy conservation requires that

2.3.

41

v=

2eV = m

2(1.6x10!19 C)(350V) = 1.11x107 m / s !31 9.11x10 kg

Since you know the velocity and magnetic field strength you know the radius of the circular trajectory.

R=

mv (9.11x10!31kg)(1.11x107 m / s) = = 0.32mm eB (1.6x10!19 C)(200mT)

The mass spectrometer is an essential tool for the identification of molecules. The molecule is blown into a set of fragments many of which are charged. The sizes of the fragments can be determined by shooting them into a magnetic field. Since the particles are charged they move in a uniform circular motion on a plane that is perpendicular to the field. The period can be used to measure the mass of the fragment:
T= 2!r 2 !m = v Be

A singly charged ion of iodine makes 7.00 revolutions in a field of 45.0 mT in 1.29 ms. Calculate its mass in grams per mole.
T= 2!r 2 !m BeT (45mT)(1.6x10#19 C)(1.29ms / 7) = "m = = = 2.111x10#25 Ns 2 / m v Be 2! 2!
= 2.111x10
!25

" 6.022x1023 % kg$ = 127g / mole mole ' # &

2.3.

42

stors shown below have the lengths and cross-sectional areas indicated and are made of material e resistivity. Which has the greatest resistance?

AP-B Synchrotron Problems magnetic filed electrons of 0.1 T forces a proton beam of 1.5 mA to move in a circle of radius to 0.1the m. the Bohr modelA of the atom, orbit the nucleus in definite orbits. According planebe of impossible the circle is perpendicular sics, this modelThe would because to the magnetic field.

laws of

ively charged nucleus the electrons (B) Coulomb's law applies 1. Of theattracts following, which is the best estimate of the work done by the magnet field on the ing electrons radiate energy (D) there is circle? a centripetal force on the electrons protons during one complete orbit of the (B) 10-22 J (C) 10-5 J (D) 102 J (E) 1020 J momentum is conserved (A) 0 J
2. Of the following, which is the best estimate of the speed of a proton in the beam as it moves in the circle? (A) 10-2 m/s (B) 10-3 m/s (C) 106 m/s (D) 108 m/s (E) 1015 m/s 3. An electron is in a uniform magnetic field B that is directed out of

l conducting spheres are charged to +2Q and of -Q. respectively, are separated a distance the plane the page, as shown.and When the electron is by moving in the d plane of the page in the direction indicated by the arrow, the force r than the radii of the spheres) as shown above. The magnitude of the force of attraction onon the left the electron is directed After the two spheres are made to touch and then are reseparated by distance d, the magnitude of (A) to the right (B) out of the page the left sphere is F2. Which of the following relationships is correct? (C) into the page (D) toward the top of the page (B) F1 = F2 (C) F1 = 2F2 (D)(E) Fl=4F (E) Fof 8 page F2 toward the 2 the bottom 1=
4. Which of the following paths above represents the path of an electron traveling without any loss of energy through a uniform magnetic field directed into the page? (A) A (B) B (C) C (D) D (E) E

paths above represents the path of an electron traveling without any loss of energy through a netic field directed into the page? (A) A (B) B (C) C (D) D (E) E

2.3.

43

2007 AP PHYSICS B FREE-RESPONSE QUESTIONS

5. Your research director has assigned you to set up the laboratorys mass spectrometer so that it will separate strontium ions having a net charge of +2e from a beam of mixed ions. The spectrometer above accelerates a beam of ions from rest through a potential difference e , after which the beam enters a region containing a uniform magnetic field B of constant magnitude and perpendicular to the plane of the path of the ions. The ions leave the spectrometer at a distance x from the entrance point. You can manually change E. Numerical values for this experiment: Strontium atomic number: 38

nts)

search director has assigned you to set up the laboratorys mass spectrometer so that it will separate Strontium ion mass: 1.45x10-25 kg m ions having a net charge of +2e from a beam of mixed ions. The spectrometer above accelerates a Magnitude of B ! field: T enters a region containing a f ions from rest through a potential difference , after which 0.090 the beam Desired exit distance x : 1.75 m m magnetic field B of constant magnitude and perpendicular to the plane of the path of the ions. The ions he spectrometer at a distance x from the entrance point. You can manually change ! .

(a) In what direction must B point to produce the trajectory of the ions shown? Use the Shapes cal values for this experiment: tools to annotate the drawing above to answer the question. (b) The ions travel at constant speed around the semicircular path. Explain why the speed 1.45 10 25 kg Strontium ion mass: remains constant.
Magnitude of B field: Desired exit distance x: 0.090 T 1.75 m Strontium atomic number: 38

what direction must B point to produce the trajectory of the ions shown?

e ions travel at constant speed around the semicircular path. Explain why the speed remains constant.

(c) Calculate the speed of the ions with charge +2e that exit at distance x. Use the equation editor to show your work.

culate the speed of the ions with charge +2e that exit at distance x.

culate the accelerating voltage ! needed for the ions with charge +2e to attain the speed you calculated (d) Calculate the accelerating voltage e needed for the ions with charge +2e to attain the speed part (c).

you calculated in part (c). Use the equation editor to show your work.

2007 The College Board. All rights reserved. 2.3.(for AP professionals) and www.collegeboard.com/apstudents (for students and parents). t apcentral.collegeboard.com

44

GO ON TO THE NEXT PAGE. -7-

2011 AP PHYSICS B FREE-RESPONSE QUESTIONS (Form B)


A constant magnetic field of magnitude B2 directed into the page is now added in region 2 to the right of region 1, as represented in the figure below. Suppose a particle leaves the radioactive source, travels through region 1 in a straight line, and enters region 2. For each of the following, express algebraic answers in terms of E, B1 , B2 , q, and fundamental constants, as appropriate.

6. A constant magnetic field of

(b)

(c) Describe the changes, if any, in the magnitude and direction of the magnetic force as the particle moves in region 2.

magnitude B2 directed into the page is now added in region 2 to the right of region 1, as represented in the figure below. Suppose a particle leaves the radioactive source, travels through region 1 in a straight line, and enters region 2. For each of the following, express algebraic answers in terms of Determine an expression for the initial magnetic force on the particle in region 2 and state its direction. E, B1, B2, q, and fundamental constants, as appropriate. for the initial magnetic force on the particle in region 2 and state its direction. Show your work equation editor. note any quantities not Derive an expression for the charge-to-mass ratio using q m of the particle. Specifically (c) Describe the changes, if any, in the magnitude and direction of the magnetic force as the particle moves in region 2. (d) Describe the path of the particle in region 2. Use the shape tools to annotate the drawing.

(b) an expression (d) Describe the pathDetermine of the particle in region 2.


(e) previously defined that are included in your answer.

(e) Derive an expression for the charge-to-mass ratio q m of the particle. Specifically note any quantities not previously defined that are included in your answer. Show your work using equation editor.

2011 The College Board. Visit the College Board on the Web: www.collegeboard.org.

GO ON TO THE NEXT PAGE. -11-

2.3.

45

2011 AP PHYSICS B FREE-RESPONSE QUESTIONS (Form B)

The diagram at the right illustrates a velocity selector, labeled region 1. It consists of two parallel conducting plates, with charges on the plates as indicated creating an electric field of magnitude E directed toward the top of the page. A uniform magnetic field of magnitude B1, directed out of the page exists between the plates. The magnitude of the magnetic field 5. so (15 points) can be adjusted that only particles of a particular speed pass through the selector in a straight line. A radioactive source to the leftillustrates of the selector emits charged particles, having the same The diagram above a velocity selector, labeled regioneach 1. It consists of two parallel conduc charge +q and moving to the right in the plane of the page. The effect of gravity can be neglected plates, with charges on the plates as indicated creating an electric field of magnitude E directed tow throughout the problem. top of the page. A uniform magnetic field of magnitude B1 directed out of the page exists between t (a) The magnitude of the magnetic field can be adjusted so that only particles of a particular speed pass i. Derive the equation E B1 line. for the speed v ofsource particles that move inselector a straight line selector in av= straight A radioactive to the left of the emits charged particles, each same charge q and moving to the to right in the plane of the page. The effect of gravity can be negle through region 1. Use the equation editor show your work.
throughout the problem. (a) i. Derive the equation ! region 1.
ii.

E B1 for the speed ! of particles that move in a straight line thro

Some particles are from theare source with speeds B1. Which ofthan the following ii. emitted Some particles emitted from thegreater source than with E speeds greater E B1 . Which of the fo describes the initial path of one of these particles immediately after entering region 1 ? entering region 1 ? describes the initial path of one of these particles immediately after ___ It curves toward the top of the page. ____ It curves out of the page. It curves toward the top ofIt the page. It curves toward the bottom of th ___ It curves toward the bottom of the page. ____ moves in a straight line. ___ It curves into the page. It curves into the page. It curves out of the page. Explain your reasoning. It moves in a straight line.

Explain your reasoning.

2011 The College Board. Visit the College Board on the Web: www.collegeboard.org.

2.3.
-10-

46 GO ON TO THE NE

The Magnetic Force on a Wire A force is exerted on a conductor containing a current in a magnetic field. A conducting wire lies on the z-axis through a region in which a magnetic field is directed along the positive y-axis. There is a force on the wire pointed along the positive x-axis. F = ev ! B = "itvk ! Bj = "iLk ! Bj = iLBi
k

(2.13)

i F B

An ampere is a Coulomb per second. We suppose that the current is steady and write

ev = !itv and tv = Lk

where t is the time interval as electrons pass through the external B field. Then tv=L is the width of the field.

We are moving towards a description of magnetic forces acting on circuit segments and so the force has been expressed in terms of the current rather than the velocity. A wire of arbitrary shape can be decomposed into small segments 'L and the force on the wire evaluated by summing over these segments. Examples A horizontal conductor in a power line carries a current of 5000 A from south to north. Earths magnetic field of 60.0 T at this location is inclined downward at 70 to the horizontal. Find the magnitude and direction of the magnetic force on 100 m of the conductor. The force is the vector product of the current and the component of the field perpendicular to the current. With the current directed north the force must point to the west with positive charge carriers. It points east if we imagine that electrons carry the current. The downward component of the field is 57T. So the magnitude of the force is iLB or 28 N.
X X X X X X X X X X X X X X X X X 62.0 cm X X X X

A current carrying wire hangs supported by two springs from X a rigid plate. The current is perpendicular to a uniform X magnetic field directed as shown. The wire has a length of 62.0 cm, a mass of 13 g, and the field strength is 0.440 T. X

What is the magnitude and direction of the current required to remove tension from the supporting springs? The magnetic force must balance the gravitational force: mg=iLB. So
i= (0.0130kg)(9.8m / s2 ) = 0.467A (0.620m)(0.440T)

2.3.

47

fish hangs at a point 1/4 of the rods length from the tackle box. What is the ma (A) 1.5 kg (B) 2 kg (C) 3 kg (D) 6 kg (E) 6.5 kg

A point P is 0.50 meter from a point charge of 5.0 X 10-3 coulomb.


(A)

IC. Problems Involving Force on a Current-Carrying Wire

44. An object swings on the end of a cord as a simple pendulum with period T. Ano down on the end of a vertical spring. also with period T. If the masses of both o new values for the Periods? Pendulum Mass on Spring

1. A wire 50 cm long lying along the x axis carries a current of 0.50 A in the +ve x direction, (B) k. T of the electric field at point P is most through a magnetic field nearly of B=3mTj + 10mT What is the magnetic force on the wire? Show (C) T T 1 2 your work using equation editor. N/C (B) 2.5 x 10 N/C (C) 9.0 x 10 N/ C (D) 1.8 x 103 N/C (E) 7.5 x 108 N/C
(D) T (E)

otential at point P is most nearly V (B) 2.5 x 101 V (C) 9.0 x 102 V

(D) 1.8x 103 V

(E) 7.5x 103 V

2. There is a counterclockwise current I in a circular loop of The scattering of alpha magnetic particles byfield a thindirected gold foil was wire46. situated in an external out measured of the by Geiger and model of the atom was proposed in order to explain why page as shown. The effect of the forces that act on this current (A) more particles scattered through angles greater than 90 than through angles is to make (B)the the loop fraction of particles scattered through large angles was too large to be ex (A) expand the in size (B) contract in size (C) rotate about an atom (C) no particles passed through the foil undeflected axis perpendicular to the page (D) rotate about an axis in the (D) the most common scattering angle was about plane of the page (E) accelerate into the page90
(E) the most common scattering angle was about 180

45. A proton collides with a nucleus of . If this collision produces a nucleus of that particle is (A) a proton (B) a neutron (C) a deuteron (D) an x par

nterclockwise current I in a circular loop of wire situated in an external magnetic field directed 3. A wire in the plane of the page carries a current directed toward the e as shown above. The effect of the forces that act on this current is to make the loop top of the page as shown above. If the wire is located in a uniform size (B) contract in size rotate about axis to the page magnetic field B(C) directed out of the an page, theperpendicular force on the wire resulting ut an axis in the plane ofmagnetic the page from the field is(E) accelerate into the page
(A) directed into the page (C) directed to the right
magnetic field is (A) directed into the page (D) directed to the left

(B) directed out of the page 47. A wire in the plane of the page carries a current directed toward the top of the pa (D) directed to the left (E) zero is located in a uniform magnetic field B directed out of the page, the force on the
(B) directed out of the page (E) zero (C) directed

48. Which of the following is always a characteristic an adiabatic process? (A) The temperature does not change ("T = 0). (B) The pressure does (C) The internal energy does not change ("U = 0). (D) No heat flows into (E) No work is done on or by the system (W = 0)

hown above, what is the value of the potential difference between points X and Y if the 6-volt internal resistance? (A) 1 V (B) 2 V (C) 3 V ( D) 4 V (E) 6V

Copyright 1988 by College Entrance Examination Board. All rights reserve College Board, Advanced Placement Program, AP, SAT, and the acorn logo are registered trademarks of the C

work is needed to move one coulomb of charge from one point to another with no change in h of the following is true between the two points? ance is one ohm. (B) The current is one ampere. ial difference is one volt. (D) The electric field strength is one newton per coulomb. c field strength is one joule per electron.

2.3.

48

Torque on a Current Loop

Is an electrical current the flow of negatively charged particles?

You saw the force and it is consistent the Lorentz force. Thats empirical. But sometimes an explanation is correct for wrong reasons. The fascinating thing about technology, for example, electrical motors and airplanes, is that even though the explanations were bogus devices built using them worked.

The force on the current segment is the basis for the electrical motor. Be an engineer when it is useful. Here that means that you are to use conventional current. This means that i is positive and the current is carried by positively charged particles. This eliminates the need to undo an application of the right-handrule. When a current loop is placed in a magnetic field there is a torque on the loop. The rectangular loop has a segment at the top with a length La. The length of the other two sides of the rectangle are 2Lb. The current loop rotates about an axis a distance Lb from each longer edge of the loop. The field produces a torque.

La B

F B i 2Lb

! top = Lb " (iLa " B)

On the bottom segment the lever arm is -Lb and the direction of the current, relative to the top segment is reversed, -La.
!bottom = "Lb # ("iLa # B) Is the torque on that segment.

!net = 2Lb " (iLa " B) is the net torque on the current loop.
When the loop rotates through ! the current in the bottom loop in the second configuration is reversed relative to the first configuration. The result is that the torque is reversed. The current loop would rotate back into the former position. To keep the torque pointed in the same direction the current direction can be reversed every half revolution of the current loop. Thats just how a motor does it.

2.3.

49

A Charge in Motion Generates a B Field So far the study of magnetic fields has involved the force on a charge in motion within a magnetic field. Now we begin to ask, how was this magnetic field produced? A charge in motion couples to a magnetic field. Then we should expect that a magnetic field can also be induced by a charge in motion. The empirical result is often attributed to Oersted but called the Biot-Savart law (pronounced Beo-Saavar).

Well see soon that it is not a law any more. We understand why it happens. James Clerk Maxwell explained it to us. It looks like a 1/r2 law, reminiscent of Coulombs law and Newton's law of universal gravitation.

If you place a compass along side a direct current the compass needle will be deflected so that it is aligned as a tangent to a circle whose origin is the wire. The four little compasses point south.

The geographic north pole of the Earth is in the vicinity of the magnetic south pole. These little compasses are showing us that the magnetic field created by the current is directed counterclockwise around the circle. The compass needles all point in the opposite direction. The direction is obtained with a right hand rule. You find the direction of the magnetic field by pointing the thumb of the right hand along the current so that the fingers point in the angular direction of the magnetic field. The current at every point along the wire contributes to every magnetic field vector. At each distance r from the current along a linear perpendicular to the wire there is a cylindrical shell of B field vectors. When contributions from all currents are summed the magnitude of the B field as a function of distance from the wire is obtained. (2.14) The magnitude of the B field is proportional to the current and inversely proportional to the perpendicular distance from the wire to the field point. The constant of proportionality is called the magnetic permeability. The value of the magnetic permeability of a vacuum, 0, is 4!x10-7 Tm/A or about 1.26x10-6 Tm/A. This may (should) remind you of the dielectric permeability, $0. Look at the units of this product of constants 2 C2 Tms C2 ms Ns " s % . [! 0 0 ] = = = $ ' 2 2
Nm C Nm C Cm # m&

2.3.

50

Parallel Currents Current i+ in the figure generates a field B+ a perpendicular d from the wire

B+ =

0i + 2!d

A current in a magnetic field experiences a force. The force on the wire with current i- is

F! = e! v ! " B+ = i! tv ! " B+ = i !l " B +


The vector along the wire, l, is taken as positive in the direction of the current. Place your thumb along the current i+ and the direction of the field B+ is pointing down along the wire carrying i- Put the right hand index finger along the current i- in the direction l and your ring finger down along the vector B+. Your thumb points away to the left; away from the parallel wire carrying the current i+. This is the direction of the force with positive, conventional current charge carriers. The force on the wire with i- acts to push it away from the wire carrying current i+. Suppose that both currents are in the same direction, i+. The force on the second wire due to the first is now directed toward the first wire. Antiparallel currents are pushed apart. Parallel currents are pulled together. Example Two long straight parallel wires, separated by a distance d, carry currents i. The one on the left is out of the plane and the one on the right into the plane.

P x

Calculate the net magnetic field at a point P above the wires and equidistant from both by a distance d/21/2. Repeat the calculation for the configuration on the right. The interior angle at the point P is 90 because the distance to P from either wire is d/21/2. The contributions to the field from current into and out of the plane are shown as vectors in the diagram at the left. One only needs to apply the right-hand-rule to the two currents. If the right hand current is out of the plane the resultant is to the left. If the right hand current is into the plane the resultant is toward the top of the screen. In both cases the magnitude is the same. And 2 0i cos(45) B= 2d!

2.3.

51

AP-B Problems on Force Between Current-Carrying Wires 1. Two long wires a distance d apart carry equal antiparallel currents i. Determine the magnetic field at a point P a distance R above the plane of the wires. Evaluate the field when d is 10 cm, R is 15 cm, and i is 10 A. Show your work using equation editor.

2. Two long, parallel wires, fixed in space, carry currents I1 and I2. The force of attraction has magnitude F. What currents will give an attractive force of 5F? (A) 2I1 and2013 I2/2 AP (B) IPHYSICS (C) I1/2 and I2/2 (D) 2I1 and 2I2 (E) 4I1 and 4I2 FREE-RESPONSE QUESTIONS 1 and I2/4 B

3. Two long, straight horizontal wires are near each other and parallel, with one directly above 6. (15 points)

Two long, straight horizontal wires are near each other and parallel, with one directly above the other as shown so that it carries a current of 65 A. Wire Y, which is part of a second circuit, is free to move in the figure. Wire X is fixed in place and connected to a battery (not shown) so that it carries a current of 65 A. vertically and suspended at circuit, rest by is the magnetic between wires. The mass permagnetic length force Y, which is is part of a second free to move force vertically and isthe suspended at rest by the Wire
3 of wirethe Y is 5.6 !The 10 mass kg m. from parts ofm the circuits that are not shown. 5.6 the 10 kg . Neglect effects from the parts of the between wires. per Neglect length ofeffects wire Y is circuits that are not shown.

the other as shown in the figure. Wire X is fixed in place and connected to a battery (not shown)
-3

(a) Calculate the magnitude of the magnetic field produced by wire X at the position of wire Y. Show your work using equation editor. field produced by wire X at the position of wire Y. (a) Calculate the magnitude of the magnetic
(b)

(b) i. Calculate the magnitude of the current in wire Y. . wire Y. ii. i. Indicate the direction of the current in wire Y Calculate the magnitude of the current in
____ To the left ____ To the right ____ Neither left nor right, since there is no current

(c) Now ii. wire Y is moved to a new position that is closer to Y wire Indicate the direction of the current in wire . X, but wire Y is still below wire X and is still carrying the same current as determined in part (b). Wire Y is released from rest. Describe the initial motion ____ To the left ____ To the right ____ Neither left nor right, since there is no current of wire Y. Justify your answer.

(c)Suppose Now wire YY isis moved position is closer , but wire Y is still wire (d) wire movedto toaanew position 0.025that m above wire to X. wire WhatX changes in current, if below any, must occur to X and is still carrying the same current as determined in part (b). Wire Y is released from rest. maintain equilibrium?
(e) With wire Y still above wire X, the circuit connected to wire Y is removed. Wire Y, which is 1.2 m long, is then moved vertically up and away from wire X at a constant speed of 3.0 m/s.

Describe the initial motion of wire Y. Justify your answer.

(d) Suppose wire Y is moved to a position 0.025 m above wire X. What changes in current, if any, ii. must occurwhich to maintain equilibrium? Indicate end of wire Y is at a higher electric potential.
____ The left end ____ The right end ____ Neither end, since they are at the same electric potential Justify your answer.

i. Calculate the magnitude of the induced emf in wire Y when the wires are 0.050 m apart.

2.3.

52

2010 AP PHYSICS B FREE-RESPONSE QUESTIONS (Form B)

4. (10 points) which in turn are connected to four resistors with resistances given in the figure above. Assume

4. In the circuit above, the battery of emf E is connected to two long, straight, parallel wires,

Inthat the any circuit above, the battery emf ! is are connected to two long, straight, parallelanswers wires, which in turn are other resistances inof the circuit negligible. Express all algebraic to the connected to parts four resistors with resistances given in the figure above. Assume that any other resistances in the following in terms of the given quantities and fundamental constants. circuit are negligible. Express all algebraic answers to the following parts in terms of the given quantities and fundamental (a) Derive constants. an expression for the total resistance of the circuit. (a) Derive an expression for the total resistance of the circuit. (b) Derive an expression for the power dissipated in this circuit.

(b) Derive an expression for the power dissipated in this circuit.

Assume that any magnetic fields result only from the currents in the two long wires. (c) What is the any direction of thefields magnetic field, if any, atthe point P, which is intwo the plane of the page? Assume that magnetic result only from currents in the long wires.

(c) What is the direction of the magnetic field, if any, at point P, which is in the plane of the ____ To the left ____ Toward the top of the page ____ Out of the plane of the page page? ____ To the left ____ Toward of the page Out of the the plane planeof ofthe the page ____ To the right ____ Toward the the top bottom of the page ____ ____ Into page ____ To the right ____ Toward the bottom of the page ____ Into the plane of the page ____ None of the themagnetic magnetic field zero ____ None of theabove, above, because because the field is is zero
Explain your reasoning. Explain your reasoning. (d) What is the direction of the force, if any, on the bottom wire due to the current in the top wire?

(d) What is the direction of the force, if any, on the bottom wire due to the current in the top wire? ____ To the right ____ Toward the bottom of the page ____ Into the plane of the page ____ To the left ____ Toward the top of the page ____ Out of the plane of the page ____ None of the above, because the the force is zeroof the page ____ Into the plane of the page ____ To the right ____ Toward bottom ____ None of the above, because the force is zero
Explain your reasoning.

____ To the left

____ Toward the top of the page

____ Out of the plane of the page

Explain your reasoning.

2010 The College Board. Visit the College Board on the Web: www.collegeboard.com.

2.3.
-8-

GO ON TO THE NEXT PAGE.

53

2008 AP PHYSICS B FREE-RESPONSE QUESTIONS (Form B)


2008 AP PHYSICS B FREE-RESPONSE QUESTIONS (Form B)

3. (15 points) A5. student is measuring the magnetic field generated by a long,by straight wire carrying a constant current. A A student is measuring the magnetic field generated a long, straight wire carrying a 3. (15 points) magnetic field probeA is magnetic held at various from wire, as shown above, and the magnetic field is constant current. fielddistances probe is d held atthe various distances d from the wire, as shown A student measured. is measuring the magnetic field generated a long, straight wire carrying a and constant current. A for the data. The graph below shows the fiveby data points the student measured a best-fit curve above, and the magnetic is measured. graph below shows the data points the magnetic field probe is held at various field distances d from theThe wire, as shown above, and thefive magnetic field 5 is Unfortunately, the student forgot about Earths magnetic field, which has a value of 5.0 10 T student measured and a best-fit curve for the data. Unfortunately, the student forgot about measured. The graph below shows the five data points the student measured and a best-fit curve for the data. at this -5 location and is directed north. 5 directed north. Earths magnetic field, which has a value field, of 5.010 atathis location and is Unfortunately, the student forgot about Earths magnetic which T has value of 5.0 10 T at this location and is directed north.

(a) On the graph, plot new points for the field due only to the wire.
(b) Calculate the value of the current in the wire.

(a) On the graph, plot new points for the field due only to the wire. Use shape tools.

(b) Calculate the value of the current in the (b) Calculate the value of the current inwire. the wire. Show your work using equation editor. (a) On the graph, plot new points for the field due only to the wire.

Another student, who does not have a magnetic field probe, uses a compass and the known value of Earths magnetic field to determine the magnetic field generated by the wire. With the current turned off, the student places the compass 0.040 m from the wire, and the compass points directly toward the wire as shown below. The student then turns on a 35 A current directed into the page.

2008 The College Board. All rights reserved. Visit apcentral.collegeboard.com (for AP professionals) and www.collegeboard.com/apstudents (for students and parents). 2008 The College Board. All rights reserved. Visit apcentral.collegeboard.com (for AP professionals) and www.collegeboard.com/apstudents (for students and parents).

2.3.

-7-7-

GO ON TO THE NEXT PAGE. 54

GO ON TO THE NEXT PAGE.

Another student, who does not have a magnetic field probe, uses a compass and the known value of Earths magnetic field to determine the magnetic field generated by the wire. With the current turned off, the student places the compass 0.040 m from the wire, and the compass points directly toward the wire as shown below. The student then turns on a 35 A current directed into the page.

(c) On the compass, sketch the general direction the needle points after the current is established.

(d) Calculate how many degrees the compass needle rotates from its initial position pointing directly north. The is part how of a circuit a power source withrotates an emf from of 120 Vinitial and negligible resistance. (d) wire Calculate many containing degrees the compass needle its positioninternal pointing (e) Calculate the total resistance of the circuit. (f) Calculate the rate at which energy is dissipated in the circuit.

(c) On the compass, sketch the general direction the needle points after the current is established. Use shape tools.

directly north. Use equation editor.

The wire is part of a circuit containing a power source with an emf of 120 V and negligible internal resistance. (e) Calculate the total resistance of the circuit. Use equation editor.

(f) Calculate the rate at which energy is dissipated in the circuit. Use equation editor.

2008 The College Board. All rights reserved. Visit apcentral.collegeboard.com (for AP professionals) and www.collegeboard.com/apstudents (for students and parents).

GO ON TO THE NEXT PAGE. -8-

2.3.

55

Amperes Law for a Solenoid The magnetic field everywhere along the circular path centered on the current-carrying wire is constant. The product of the length of the path and the magnitude of the magnetic field is proportional to the current.

This result was generalized by Ampere. Identify a path through which a current passes and along which the magnetic field has a constant magnitude. The product of the length of this path and the magnitude of the field is proportional to the current enclosed. path length magnetic field strength = = ienclosed The solenoid is an important symmetry in which a current-carrying wire is wrapped into a cylinder. Many devices use solenoids, such as guitar pickups, starter motors, bank vaults. Solenoids are circuit elements called inductors that are used to make wireless communication systems and radio transmitters. Inside the cylinder the magnetic field produced by the current is very uniform. Outside the solenoid the magnetic field vanishes. In the part of the field that is nonzero the rectangular path shown has a length L. The path encloses N wrappings, each carrying the same current. So if the interior of the solenoid is just empty space Amperes rule says (2.15) Here n is the linear density of wrappings, n= N/L. The direction of the field is in the direction of the thumb of your right hand when the fingers curl in the direction of the current in the wrappings. To reverse the direction of the field the direction of the current is reversed. To increase the strength of the field for a particular current the interior is filled with a material with a larger magnetic susceptibility. Examples A 200 turn solenoid having a length of 25 cm and a diameter of 10 cm carries a current of 0.3 A. Calculate the magnitude of the magnetic field inside the solenoid assuming that it behaves ideally. "7
B= 0Ni (4 !x10 Tm / A)200(0.30A) = = 0.3mT L 0.25m

2. An electromagnet is created by inserting a ferromagnetic material in the solenoid. The figure at the right shows possible magnetic fields for the space between the poles of a horseshoe-shaped magnet. Use Amperes rule we see that at the edge they cannot be correct. The path shown contains no current. So the field lines at the edges must be bent. This is called fringing.

2.3.

56

IC. Problems Involving Solenoids 1. A 200 turn solenoid having a length of 25 cm and a diameter of 10 cm carries a current of 0.29 A. (a) Calculate the magnetic filed B inside the solenoid. (b) What is the magnetic dipole moment of the solenoid?

2. A solenoid 1.30m long and 2.60 cm in diameter carries a current of 18.0A. The magnetic field inside the solenoid is 23.0 mT. Find the length of the wire forming the solenoid?

3. A long solenoid has 100 turns/cm and carries current i. An electron moves within the solenoid in a circle of radius 2.30 cm perpendicular to the solenoid axis. The speed of the electron is 0.0460c (c = speed of light). Find the current i in the solenoid.

4. Using the magnetic field sensor, some wire and a pencil, confirm Amperes rule.

2.3.

57

Magnetic Imaging Consider these case posts: Magnetoencephalography and SQUIDS in vivo Magnetic Resonance Spectroscopy Case Rubric Opportunities to construct a summary of an Engineering Case will be provided throughout the year. Each student must post two cases. Your summaries are no more than 5 MByte. They should be posted on techphysics.org. The summary should include: Statement defining the problem addressed by the design (20 pts) Statement defining the essential features of the design (20 pts) If the design or technology failed, what causes were identified? (30 pts) or If the design or technology succeeded, what were the consequences? (30 pts) or If sequence is considered, how has the design or technology evolved? (30 pts) Sources used in your review are cited and credible (10 pts) The work is free of grammatical and spelling errors (20 pts) Each student must review five posts. The review is posted at the site with the document reviewed. Your review is 300 words or less. The reviews should include: Identify author and title of work reviewed (5 pts) Criticism is constructive and well-reasoned (20 pts)

2.3.

58

Flux A small net is used to clean a fish tank. The rim of the net is swept through a volume V. The rate of flow of water into the net through the area bounded by the rim is equal to the rate of flow of water out through the area of the net attached to the rim. This volume rate of flow is a flux. This example provides a common strategy. If you wanted to know the flux through the net measurements over a complicated surface are needed. However, measurements over the circular rim are easy. The flux of water out through the net must equal the flux of water in through the rim. The total flux of water through the closed surface is zero because no water is being created or destroyed: water that goes in must come out. The flow rate of water in through the ring is simple because the geometry of the ring is simple and the rate of flow of water through the ring is determined by the motion of the ring, which can be described simply Flux of electric or magnetic fields through a surface occurs when the field line cross the surface. Only the part of the field perpendicular to the surface contributes to the flux. Suppose that the square here is 3.2 mm on each side. It is immersed in a uniform electric field with a magnitude of 1800 N/C. The field lines make an angle of 35 with the surface normal. Calculate the electric flux through the surface. Let e be a unit vector aligned with the parallel electric field lines. And let the surface normal lie along the x axis of a Cartesian coordinate system. Then E=1800N/C e and (ie)=cos(35) so the flux is equal to 1800 N/C (3.2x10-3 m)2 cos(35). The symbol (E is used to denote the electric field flux. (E=0.15Nm2/C. Since an alternative unit for the field is the volt per meter the electric field flux can also be expressed in volt meter. The symbol (B is used to denote the magnetic field flux. The magnetic flux has its own SI unit, the Weber: 1 Weber = 1Tesla m2. The sketch at the left describes the basic structure of an electric motor. The magnetic field lines are parallel and pointing into the page. The magnetic flux through the wire loop determines the output from the motor. Suppose the magnetic field strength is 0.15T and that the loop is perpendicular to the field. The value of a is 10 cm and the value of b is 3 cm. Then (B=0.15 T (0.1m)(0.03m)=4.5 mW. When the plane of the loop is parallel to the field the flux is zero.

2.3.

59

Faraday Induction A charge generates an electric field. A charge in motion generates a magnetic field.
#$$

This means that there must be a way of describing electric and magnetic fields that includes this dependence on relative motion. This representation was obtained by Maxwell and from it our description of light. His work was made possible by Faradays observation of the current induced in a solenoid by a changing magnetic field flux. When a permanent magnet moves through a solenoid, current flows in the wires of the coil. If there is a current then there must be an EMF. Faradays empirical law relates the EMF, E, to the time dependence of the magnetic field. When a magnet moves through a solenoid the magnetic field flux, (B, changes with time. In an electric circuit where there is an EMF there is a potential drop, for example over the poles of a battery. Recall that 'V=Vf-Vi <0, while the EMF which drives the current is positive. The pickup on an electric guitar is a solenoid wrapped around a permanent magnet. When the metal string above the pickup vibrates the magnetic field within the magnet changes. When the field changes current is produced. You induce a current in your Fender pickup by waving it in front of your Marshall amplifier. The sound waves from the amp cause the strings to vibrate. This could further increase the current in your pickup which can cause the amp to scream. This could cause the strings to vibrate more which could cause the amp to scream louder and so on. That's positive feedback for you. But that doesnt happen. We get feedback but the feedback is negative. The Universe is stable and induction effects involve negative feedback. It's called Lenz's law. It is just the negative sign in Faraday's law. The direction of the changing flux determines the direction of current induced. Here is how you can use your right hand rule to determine the direction of current. Point your thumb in the direction in which the flux is increasing. Then rotate your thumb to point in the opposite direction. Your fingers curl in the direction of the induced current.

!&" %&"

'$$

But in motion means with respect to some coordinate frame. Let the coordinate frame be the body-fixed frame of the charged particle. Only an % " electric field remains!

! "

2.3.

60

Field and EMF Examples Expanding Hoop k Thermal expansion of a conducting ring produces a current in the presence of a magnetic field. Consider a ring of conducting material in which there is a counterclockwise current. There is a uniform magnetic field pointing along the positive k axis. Using the fingers of your right hand show that there is a force everywhere on the ring that will cause it to expand. What is the direction of the induced magnetic field when the loop expands? As the area increases the field flux through the area increases. The direction of the field is upward so you put your thumb along k. To determine the direction of the induced current you flip your hand over so that your thumb points down. The fingers of your right hand now curl in the clockwise direction. The induced current direction is clockwise. Using the fingers of your right hand show that the force exerted on the loop due to the induced current will cause the ring to contract. Magnetic Switch An EMF can be induced in a conductor by changing the magnetic field flux. You can change the flux by changing the magnetic field strength. You can change the flux by changing the direction of the magnetic field relative to the cross-sectional area of the conductor. For example, you could let a wire loop rotate in a fixed magnetic field. This is what the electrical motor does. You can change the flux by changing the cross-sectional area of the conductor. This is what a magnetic switch does. A rectangular loop of wire, oriented perpendicular to the external magnetic field makes the analysis simple. In this picture the loop is being pushed into a magnetic field directed into the plane of the page. This means that the flux is increasing in the direction into the page. You point your thumb in the direction opposite to the increasing flux and your get an induced current that is counterclockwise. Imagine pulling the wire loop out of the field. The flux through the loop is decreasing. Since the field vector is into the page the change of flux is negative into the page. Then the change of flux is positive out of the page. Using your right hand show that the direction of the current is clockwise. Suppose that in these examples the wire loop is moving with a constant speed v and that the short side of the loop has a length L. Then the cross-sectional area in the field is changing 'A/'t=Lv. Then the changing magnetic flux is '(B/'t=BLv.

2.3.

61

Heat Release in an Inductor A 50.0 cm of copper wire with a diameter of 1.00 mm is formed into a circular loop and placed perpendicular to a uniform magnetic field that is increasing at the constant rate of 10 mT/s. The resistivity of copper is 17nWm. At what rate is heat energy produced? The changing magnetic field produces an EMF. The EMF produces a current. When current is flowing through a wire the temperature of the wire increases and heat is transported as radiant energy.
R=! The resistance is a property of the dimensions of the wire. We can assume that A Ohms approximation works for a copper wire. The power dissipated through a resistor is proportional to the voltage times the current or current squared times the resistance. L

Recall that a volt is a Joule/Coulomb. So Vi has units of Watts. Then removing i we get V2/R. Since the EMF is proportional to the rate of change of flux we square that and divide by the resistance.
1 # d" & 1 # dB& A 3 # dB& %! ( = %!A ( = % ( R $ dt ' )L / A $ dt ' )L $ dt '
2 2 2

Heat is transported at a rate of (0.5 mm)2!)3/17nWm/0.5m(10mT/s)2 = 4W. Rate of a Changing Flux Suppose that the time dependence of the flux is specified; (B=6t. Here the flux is in nanowebers and t is in seconds. What is the magnitude and direction of induced in loop of copper wire with a resistance of 2 % immersed in the field? The EMF is -6mW/s and the direction of the current induced produces a magnetic field that opposes the direction of increasing flux. If the flux is increasing along k and the loop is in the xy plane, then the current is clockwise. The magnitude of the current is 6nW/s/2 % = 3mA.
X X X X X X X X X X X X X X X X

A square wire loop with 2.0 m sides is perpendicular to a uniform magnetic field, with half the area of the loop in the field. The loop X X contains a 20 V battery with negligible internal resistance. The X X magnitude of the field varies with time according to B=0.0420 0.870t, 20V with B in teslas and t in seconds.
X X X X

What is the total EMF in the circuit? What is the direction of the current through the battery? Use your head and hands to work out these answers. The induced EMF in the loop will be in a direction that produces a current in the same direction. There is a clockwise current as you look at the loop. There is an induced EMF is in the same direction as the 20V cell potential. The total EMF is 20V+1.74V=22V.

2.3.

62

which circuit should the battery be connected to obtain the greatest steady power dissipation? h circuit will retain stored if the battery is connected to it and then disconnected? AP-B Fluxenergy Problems
1. The figure at the left shows a rectangular loop of wire of width l and resistance R. One end of the loop is in a uniform magnetic field of strength B at right angles to the plane of the loop. The loop is pulled to the right at a constant speed v. What are the magnitude and direction of the induced current in the loop?

igure above shows a rectangular of wire of width l and resistance R. One end of the loop is in a Magnitude loop Direction (A) BlvR Clockwise rm magnetic field of strength B at right angles to the plane of the loop. The loop is pulled to the right at a (B) BlvR ant speed v. What are the magnitudeCounterclockwise and direction of the induced current in the loop? (C) Blv/R Clockwise Magnitude Direction (D) Blv/R Counterclockwise BlvR Clockwise (E) 0 Undefined BlvR Counterclockwise Blv/R Clockwise 2. A rectangular wire loop is at rest in a uniform magnetic field Blv/R Counterclockwise B of magnitude 2T that is directed out of the page. The loop 0 Undefined

measures 5 cm by 8 cm, and the plane of the loop is perpendicular to the field, as shown. The total magnetic flux kilogram block of metal with a specific heat of 100 joules per kilogram*Kelvin falls from rest through a through the loop is -3 2 nce of 100 meters to the Earth's surface. If half of the potential energy by the (A) zero (B) 2x10 Tmlost (C) fallen 8x10-3 block Tm2 is converted -1 2 -1 2 ernal energy of the block, the temperature change of the block is most nearly (D) 2x10 Tm (E) 8x10 Tm

(B) 5 K

(C) 10 K

(D) 25 K

(E) 45 K

Before cable people used antennas and television was free ... you just had to endure commercials. with cable you pay for entiretemperatures channels that are But maximum efficiency of a heat Now engine that operates between of nothing 1500 Kbut in commercials. the firing chamber you have many more channels to choose from. The radio waves sent by radio and TV stations are 600 K in the exhaust chamber is most nearly (A) 33% (B) 40% (C) 60% ( D) 67% (E) 100% electromagnetic radiation .. light. And light is a combination of electric and magnetic fields. Well be there soon.

eal gas is made up of N diatomic molecules, each of mass M. All of the following statements about this gas ue EXCEPT: 3, A UHF television circular loop antenna has a diameter of 11 cm. The magnetic field of a TV The temperature of the gas is proportional average kinetic energy of the molecules. signal is normal to the plane to of the the loop and, translational at one instant, is changing at a rate of 0.16T/s. If the have field is uniform the antenna loop, what is the magnitude of the EMF in the antenna? All of the molecules the same within speed. The molecules make elastic collisions with the walls of the container. (A) 0 V with (B) 61 V (E).1.5 mV The molecules make elastic collisions each other. (C) 6.1mV (E).15 V he average number of collisions per unit time that the molecules make with the walls of the container epends on the temperature of the gas. s 33-34

2.3.

63

4. Two concentric circular loops of radii b and 2b made of the same type of wire, lie in the plane of the page as shown. The total resistance of the wire loop of radius b is R. What is the resistance of the wire loop of radius 2b? (A) R/4 (B) R/2 (C) R (D) 2R (E) 4R 5. A uniform magnetic field B that is perpendicular to the plane of the page now passes through the loops, as shown. The field is confined to a region of radius a, where a<b, and is changing at a constant rate. The induced emf in the wire loop of radius b is E. What is the induced emf in he wire loop of radius 2b? (A) Zero (B) E/2 (C) E (D) 2 E (E) 4 E 6. If the current in the loops of radius 2b is i2b and the current in the loop with radius b is ib, then the ratio i2b/ib is equal to (A) One (B) Two (C) Four (D) One half (E) One fourth 7. A single circular loop of wire in the plane of the page is perpendicular to a uniform magnetic field B directed out of the page, as shown. If the magnitude of the magnetic field is decreasing, then the induced current in the wire is (A) directed upward out of the paper (B) directed downward into the paper (C) clockwise around the loop (D) counterclockwise around the loop (E) zero (no current is induced) 8. A metal spring has its ends attached so that it forms a circle, as shown at the right. It is placed in a uniform magnetic field. Which of the following will NOT cause a current to be induced in the spring? (A) Changing the magnitude of the magnetic field (B) Increasing the diameter of the circle by stretching the spring (C) Rotating the spring about a diameter (D) Moving the spring parallel to the magnetic field (E) Moving the spring in and out of the magnetic field 9. A square loop of copper wire is initially placed perpendicular to the lines of a constant magnetic field of 5 x 10-3 tesla. The area enclosed by the loop is 0.2 square meter. The loop is then turned through an angle of 90 so that the plane of the loop is parallel to the field lines. The turn takes 0.1 second. The average emf induced in the loop during the turn is (A)1.0x10-4 V (B)2.5x10-3 V (C)0.01V (D)100V (E)400V

2.3.

64

(C) The same as the original speed (E) Four times the original speed

(D) Twice the original speed

52. A person pushes a box across a horizontal surface at a constant speed of 0.5 meter per second. The box has a mass of 40 kilograms, and the coefficient of sliding friction is 0.25. The power supplied to the box by the person is (A) 0.2 W (B) 5 W (C) 50 W (D) 100 W (E) 200 W 10. In each of the following situations, a bar magnet is aligned along the axis of a conducting loop. The magnet and the loop move with the indicated velocities. In which situation will the bar 53. In each of the following situations, a bar magnet is aligned along the axis of a conducting loop. The magnet and Questions 45-46 magnet NOT induce a current in the conducting loop? the loop move with the indicated velocities. In which situation will the bar magnet NOT induce a current in the conducting loop?

Two large, flat, parallel, conducting plates are 0.04 m apart, as shown above. The lower plate is at a potential of 2 V with respect to ground. The upper plate is at a potential of 10 V with respect to ground. Point P is located 0.01 m above the lower plate. 45. 46. The electric potential at point P is a. 10 V b. 8 V c. 6 V d. 4 V e. 2 V

The magnitude of the electric field at point P is a. 800 V/m b. 600 V/m c. 400 V/m d. 200 V/m e. 100 V/m 54. A parallel-plate capacitor has a capacitance Co. A second parallel-plate capacitor has plates with twice the area and twice the separation. The capacitance of the second capacitor is most nearly (A) !Co (B) "Co (C) Co (D) 2Co (E) 4Co

55. A vibrating tuning fork sends sound waves into the air surrounding it. During the time in which the tuning fork makes one complete vibration, the emitted wave travels 11. Two conducting wire move near a very long, straight conducting wire that carries a (A) one wavelength (B) about 340loops meters 47. Two conducting wire loops nearshown a very above, long, straight conducting wire that carries a current I. When the loops are in themove positions (C) a distance directly proportional to the frequency of the vibration they are moving in the directions current I. When the loops are in the shown they are moving in the directions shown with the same constant speed vroot . positions Assume the above, loops are far enough apart that they do (D) a distance directly proportional to the square of the that air density shown with the same constant speed v . Assume that the loops are far enough apart that theyin not affect each other. Which of the following is true about the induced electric currents, if any, (E) a distance inversely proportional to the square root of the pressure do not affect each other. Which of the following is true about the induced electric currents, if loops? 56. Two planetsthe have the same size, but different masses, and no atmospheres. Which of the following would be the any,Loop in the l loops? Loop 2 same for objects with equal mass on the surfaces of the two planets? (A) No current No current Loop l Loop 2 Counterclockwise direction (B) No current current direction No current (C) a. No Clockwise No current b. No current Counterclockwise (D) Clockwise direction Clockwise direction direction directiondirection No current (E) c. Clockwise Counterclockwise Clockwise direction d. Clockwise direction Clockwise direction e. Counterclockwise direction Clockwise direction

2.3.
Copyright 2004 by College Entrance Examination Board. All rights reserved. Visit apcentral.collegeboard.com (for AP professionals) and www.collegeboard.com/apstudents (for AP students and parents)

65

2010 AP PHYSICS B FREE-RESPONSE QUESTIONS

6. (10 points) 12. The plastic cart shown in the figure above has mass 2.5 kg and moves with negligible friction on plastic a horizontal surface. Attached to the cart is a rigid rectangular loop of wire friction that is 0.10 m by The cart shown in the figure above has mass 2.5 kg and moves with negligible on a horizontal surface. Attached to the cart is a rigid rectangular loop of wire that is 0.10 m by 0.20 m, has resistance 4.0 !, 0.20 m, has resistance 4.0 W, and has a mass that is negligible compared to the mass of the cart. and has a mass is negligible loop compared to the to mass the cart. The planeA ofuniform the rectangular loop is parallel The plane of that the rectangular is parallel theof plane of the page. magnetic field of to the plane of the page. A to uniform magnetic field ofplane 2.0 T,of perpendicular to and into the plane of the page, 2.0 T, perpendicular and directed into the the page, starts at directed x = 0, as shown above. starts at x the = 0,figure as shown above. (a) On below, indicate the direction of the induced current in the loop when its front (a) On the figure below, indicate the direction of the induced current in the loop when its front edge is at x 0.12 m.

edge is at x = 0.12 m. Annotate the figure using shape tools. Justify your answer.

your (b)Justify When the answer. front edge of the rectangular loop is at x = 0.12 m, its speed is 3.0 m s . Calculate the following for that instant. Use equation editor. (b) When the front edge of the rectangular loop is at x 0.12 m, its speed is 3.0 m s . Calculate the following i. The magnitude of the induced current in the rectangular loop of wire for that instant. i. The magnitude of the induced current in the rectangular loop of wire ii. ii. The magnitude of the on the loop The magnitude of net theforce net force on the loop (c) At a later time, the cart and loop are completely inside the magnetic field. Determine the magnitude of the net force on the loop at that time. Justify your answer.

(c) At a later time, the cart and loop are completely inside the magnetic field. Determine the magnitude of the net force on the loop at that time. Justify your answer.

2010 The College Board. Visit the College Board on the Web: www.collegeboard.com.

2.3.
-12-

66 GO ON TO THE NEXT PAGE.

2009 AP PHYSICS B FREE-RESPONSE QUESTIONS

13. A metal rod of mass 0.22 kg lies across two parallel conducting rails that are a distance of 3. (15 points) 0.52 m apart on arod tabletop, askg shown intwo the top conducting view above. Aare 3.0 W resistor isapart connected across A metal of mass 0.22 lies across parallel rails that a distance of 0.52 m on a ! resistor is connected across the left ends of the rails. The tabletop, as rails. shown in the top view above. A 3.0 the left ends of the The rod and rails have negligible resistance but significant friction with rod and rails have negligible resistance but significant friction with a coefficient of kinetic friction of 0.20. a coefficient of kinetic friction of 0.20. There is a magnetic field of 0.80 T perpendicular to the There is a magnetic field of 0.80 T perpendicular to the plane of the tabletop. A string pulls the metal rod to the right with aA constant speed of 1.8 m/s.metal rod to the right with a constant speed of 1.8 m/s. plane of the tabletop. string pulls the (a) Calculate the magnitude of the current induced in the loop formed by the rod, the rails, and the resistor.(a) Calculate the magnitude of the current induced in the loop formed by the rod, the rails, and the resistor.
(b) Calculate the magnitude of the force required to pull the rod to the right with constant speed. (c) Calculate the energy dissipated in the resistor in 2.0 s. (d) Calculate the work done by the string pulling the rod in 2.0 s. (e) Compare your answers to parts (c) and (d). Provide a physical explanation for why they are equal or (b) Calculate the magnitude of the force required to pull the rod to the right with constant speed. unequal.

(c) Calculate the energy dissipated in the resistor in 2.0 s.

(d) Calculate the work done by the string pulling the rod in 2.0 s.

(e) Compare your answers to parts (c) and (d). Provide a physical explanation for why they are equal or unequal. 2009 The College Board. All rights reserved.
Visit the College Board on the Web: www.collegeboard.com.

GO ON TO THE NEXT PAGE. -8-

2.3.

67

of 16 V. The wire has resistivity 1.7 10 8 !m and cross-sectional area 3.5 10 9 m 2 . When the power supply is turned on, the current in the wire is 4.0 A.

(a) Calculate the length of wire used to make the loop.

The wire loop is then used in an experiment to measure the strength of the magnetic field between the poles of a magnet. The magnet is placed on a digital balance, and the wire loop is held fixed between the poles of the magnet, as shown below. The 0.020 m long horizontal segment of the loop is midway between the poles and perpendicular to the direction of the magnetic field. The power supply in the loop is turned on, so that the 4.0 A current is in the direction shown.

14. A rectangular wire loop is connected across a power supply with an internal resistance of 0.50 W and an emf of 16 V. The wire has resistivity 1.7 108 Wm and cross-sectional area 3.5 109 m2 . When the power supply is turned on, the current in the wire is 4.0 A.
Note: Figure not drawn to scale.

(a) Calculate the length of wire used to make the loop.

(b) In which direction is the force on the magnet due to the current in the wire segment? ____Upward Justify your answer. ____Downward

(c) The reading on the balance changed by 0.060 N when the power supply was turned on. Calculate the strength of the magnetic field.

The wire loop is then used in an experiment to measure the strength of the magnetic field between the poles of a magnet. The magnet is placed on a digital balance, and the wire loop is held fixed between the poles of the magnet, as shown below. The 0.020 m long horizontal segment of the loop is midway between the poles and perpendicular to the direction of the magnetic field. The power supply in the loop is turned on, so that the 4.0 A current is in the direction shown. (b) In which direction is the force on the magnet due to the current in the wire segment? ____Upward ____Downward Justify your answer.

2008 The College Board. All rights reserved. Visit apcentral.collegeboard.com (for AP professionals) and www.collegeboard.com/apstudents (for students and parents).

(c) The reading on the balance changed by 0.060 N when the power supply was turned on. ON TO THE NEXT PAGE. Calculate the strength of the GO magnetic field.
-7-

2.3.

68

Suppose that various rectangular loops with the same total length of wire as found in part (a) were constructed such that the lengths of the horizontal segments of the wire loops varied between 0.02 m and 0.10 m. The horizontal segment of each loop was always centered between the poles, and the current in each loop was always 4.0 A. The following graph represents the theoretical relationship between the magnitude of the force on the 2008 AP PHYSICS B FREE-RESPONSE QUESTIONS magnet and the length of the wire.

Suppose that variousbetween rectangular0.02 loopsm with the 0.10 same total length of wire as found in part of (a) were and m. The horizontal segment eachconstructed loop was always centered between such that the lengths of the horizontal segments of the wire loops varied between 0.02 m and 0.10 m. The the poles, and the current in each loop was 4.0 A. The horizontal segment of each loop was always centered between the poles, and always the current in each loop following was always graph represents the relationship between the magnitude of theof force on on the magnet and the length of the 4.0 A. The followingtheoretical graph represents the theoretical relationship between the magnitude the force the magnet and the length of the wire. wire.

Suppose that various rectangular loops with the same total length of wire as found in part (a) were const such that the lengths of the horizontal segments of the wire loops varied between 0.02 m and 0.10 m. Th 2008 AP PHYSICS B FREE-RESPONSE horizontal QUESTIONS of each loop was always centered between the poles, and the current in each loop wa 15. Suppose that various rectangular loops withsegment the same total length of wire as found in part (a) 4.0 A. The following graph represents the theoretical relationship between the magnitude of the force on were constructed such that the lengths of the horizontal segments of the wire loops varied magnet and the length of the wire.

(d)

(d) On the graph, sketch a possible relationship between the magnitude of the On the graph above, sketch a possible relationship between the magnitude of the force on the magnet and the force on the magnet and the length of the length of the wire segment if the wire segments were misaligned and placed at a constant nonperpendicular angle to the magnetic field, as shown below. wire segment if the wire segments were misaligned and placed at a constant nonperpendicular angle to the magnetic field, (d) On the graph above, sketch a possible relationship between the magnitude of the force on the magne as shown below. Use shape tools. length of the wire segment if the wire segments were misaligned and placed at a constant nonperpen
angle to the magnetic field, as shown below.

(e) Suppose the loops are correctly placed perpendicular to the field and the following data are obtained. Describe a likely cause of the discrepancy between the data and the theoretical relationship.

(d) On the graph above, sketch a possible relationship between the magnitude of the force on the magnet and the length of the wire segment if the wire segments were misaligned and(e) placed at athe constant nonperpendicular Suppose loops are correctly placed are perpendicular to the field and the following data are obtained Suppose the loops correctly placed angle to the magnetic field, as shown below. Describe a likely cause of the discrepancy between the data and the theoretical relationship.

perpendicular to the field and the data at the left are obtained. Describe a likely cause of the discrepancy between the data and the theoretical relationship.

(e) Suppose the loops are correctly placed perpendicular to the field and the following data are obtained. Describe a likely cause of the discrepancy between the data and the theoretical relationship.

2008 The College Board. All rights reserved. Visit apcentral.collegeboard.com (for AP professionals) and www.collegeboard.com/apstudents (for students and parents).

GO ON TO THE NEXT PAGE. -8-

2008 The College Board. All rights reserved. Visit apcentral.collegeboard.com (for AP professionals) and www.collegeboard.com/apstudents (for students and parents

GO ON TO THE NEXT -8-

2008 The College Board. All rights reserved. Visit apcentral.collegeboard.com (for AP professionals) and www.collegeboard.com/apstudents (for students and parents).

GO ON TO THE NEXT PAGE. -8-

2.3.

69

Antennas Consider these case posts: Antenna design Integrated circuit Light Emitting Diode Case Rubric Opportunities to construct a summary of an Engineering Case will be provided throughout the year. Each student must post two cases. Your summaries are no more than 5 MByte. They should be posted on techphysics.org. The summary should include: Statement defining the problem addressed by the design (20 pts) Statement defining the essential features of the design (20 pts) If the design or technology failed, what causes were identified? (30 pts) or If the design or technology succeeded, what were the consequences? (30 pts) or If sequence is considered, how has the design or technology evolved? (30 pts) Sources used in your review are cited and credible (10 pts) The work is free of grammatical and spelling errors (20 pts) Each student must review five posts. The review is posted at the site with the document reviewed. Your review is 300 words or less. The reviews should include: Identify author and title of work reviewed (5 pts) Criticism is constructive and well-reasoned (20 pts)

2.3.

70

RL Circuit A changing magnetic field flux within a solenoid generates an EMF. But the EMF induced changes the current that changes the magnetic field flux. This is similar to the behavior of a capacitor when the charge on the plates of the capacitor change the voltage across the plates which then causes a change in the rate at which the plates charge or discharge. Many processes show this kind of behavior. The one that may be easiest to understand is the rate at which human population increases. As the number of people increases the rate at which new people are added to the population increases.
!Npeople !t = cNpeople

For the RC circuit the proportionality constant is 1/RC and When a solenoid is put into a circuit with a battery the rate of change of current in the solenoid is proportional to the current. A characteristic of the solenoid called inductance defines the proportionality constant. (2.16) The circuit is called the RL circuit. The RL circuit is the combination from which radio transmitters, guitar pickups, and wifi are constructed. Like the resistor, the inductor dissipates energy provided by the battery. Radiant energy is emitted from the circuit through both the resistor and the inductor. A third combination of circuit elements will show how changing fluxes in the electric and magnetic fields are connected. This third circuit is called the LC circuit and from it we learn how light shines through a vacuum. The capacitor gets charged when the switch is closed and the inductor is out of the loop. Then the switch is thrown to the position shown that isolates the LC pair. The plate of the capacitor that is initially positive produces a clockwise current. The inductor generates a self-induced EMF and the current begins to drop. When the current has reached its maximum value the capacitor is fully discharged. But the other plate of the conductor is now fully charged the current has dropped to zero. Then the current reverses, as does the process. The LC circuit is behaving as a harmonic oscillator with an angular speed of 1/ LC in which the charge on the capacitor and the current in the inductor are exactly out of phase:
q = aE sin( t LC )

and

i=

aB LC

cos(

t LC

If the current and charge are sinusoidal then E field (q/$0A) and B field (0ni) must be also. E = E0 sin(! t ) and B = B0 cos(! t ) with E0 = ! B0

2.3.

71

The RLC Circuit Walking around a circuit composed of all three elements q "i E ! iR ! ! L = 0 C "t Multiplying through by the current. Each term corresponds to a different process of energy transformation! Its a beautiful circuit.
!qE !t

is the rate at which the chemical energy is being released is the rate at which heat energy is being released is the rate at which energy is being stored at the capacitor in an electric field is the rate at which energy is being stored at the inductor in a magnetic field

i2R
q !q C !t Li !i !t

By summing the rates at which energy is stored in the capacitor, UE, and stored in the inductor, UB, expressions for the maximum energies are obtained (calculus required). These expressions can be evaluated using the capacitance of a parallel plate capacitor and the inductance of a solenoid.
UB = Li2 Li B NBA B B2 = = = hA 2 2 0n 2 0n 2 0

and

UE =

q2 CV 2 Cd2 2 ! 0 Ad 2 = = E = E 2C 2 2 2

The magnetic field energy per unit volume is just B2/20. The electric field energy per unit volume is just $0E2/2. If the resistor and battery are removed the distribution of charge in the LC circuit.

We see that the maximum value of the energy stored in the capacitor is equal to the maximum value of the energy stored in the inductor. And the ratio of the maximum electric field energy per unit volume, $0E2/2, to the maximum magnetic field energy per unit volume, B2/20, is equal to one and the angular speed of the oscillating fields has a special value.
UE E2 1 = ! 0 0 2 = ! 0 0" 2 = 1 # " = = c = 299,792,458 m / s UB B ! 0 0

Electromagnetic radiation (light) is a pair of oscillating electric and magnetic fields that conserves energy and travel at a speed c. This is where we ended the 19th century. In another 20 years physics would be transformed with the discovery that the continuous light wave was also a discrete (quantized) packet of energy called the photon.

2.3.

72

Wireless Communications Consider these case posts: G (generation) designations SIM, IMSI, and E.164 Base stations and line-of-sight propagation Near field communications Case Rubric Opportunities to construct a summary of an Engineering Case will be provided throughout the year. Each student must post two cases. Your summaries are no more than 5 MByte. They should be posted on techphysics.org. The summary should include: Statement defining the problem addressed by the design (20 pts) Statement defining the essential features of the design (20 pts) If the design or technology failed, what causes were identified? (30 pts) or If the design or technology succeeded, what were the consequences? (30 pts) or If sequence is considered, how has the design or technology evolved? (30 pts) Sources used in your review are cited and credible (10 pts) The work is free of grammatical and spelling errors (20 pts) Each student must review five posts. The review is posted at the site with the document reviewed. Your review is 300 words or less. The reviews should include: Identify author and title of work reviewed (5 pts) Criticism is constructive and well-reasoned (20 pts)

2.3.

73

Anda mungkin juga menyukai